Сохранен 513
https://2ch.hk/spc/res/421803.html
Домены arhivach.top и arhivach.site временно не функционируют! Используйте домен ARHIVACH.XYZ.
24 декабря Архивач восстановлен после серьёзной аварии. К сожалению, значительная часть сохранённых изображений и видео была потеряна. Подробности случившегося. Мы призываем всех неравнодушных помочь нам с восстановлением утраченного контента!

Тред тупых вопросов №96 CASSINI EDITION

 Аноним 11/09/18 Втр 08:46:45 #1 №421803 
89772492133918518480.jpg
20171005cassini-last-dance-regan-annotated.png
ScEh77e.jpg
videoplayback.webm
Тред вопросов о жизни, Вселенной и всем таком.

Спрашиваем то, за что в других местах выдают путёвку в биореактор. Здесь анонимные ученые мирового уровня критически рассмотрят любые гениальные идеи и нарисованные в Paint схемы.

Предыдущий тут: >>417491 (OP)
https://2ch.hk/spc/res/417491.html

Q: Можно быстрее?
A: Можно упасть в пузырь альбукерке, наса уже почти надула его.

Q: Я начитался охуительных историй про уфологию, че делать, нам жопа?
A: Да, тебе жопа, можешь сгонять в зогач или куда оттуда пошлют.

Q: Что будет с человеком в вакууме без скафандра / если он упадет на черную дыру / попробует ступить на поверхность газового гиганта/солнца?
A: Он умрёт.

Q: Почему бы не привязать ракету к воздушному шару или стартовать с горы?
A: Космос - это не как высоко, а как быстро, большая часть энергии ракеты уходит на разгон вбок.
Подробнее тут https://what-if.xkcd.com/58/ (английский) https://chtoes.li/orbital-speed/ (перевод)
Аноним 11/09/18 Втр 08:49:38 #2 №421806 
you said what.jpg
>>421803 (OP)
Ебанный стыд...
Во-первых, Алькубьерре.
Во-вторых, не упасть, а создавать вокруг корабля изнутри (иначе кина не будет).
В-третьих, НАСА искривляет пространство на десятимиллионную часть, контролируя это сверхточными интерферометрами, до самого варп-привода здесь - как до Антарктиды раком.
Аноним 11/09/18 Втр 08:50:57 #3 №421808 
>>421806
Ты с утра сидел наготове и ждал переката, что ли?
Аноним 11/09/18 Втр 08:52:17 #4 №421809 
>>421808
У меня весь спейсач открыт, я по нотификации вижу когда что.
Люблю раздел. Он мой родной самый.

Кассини это отличная тема переката, я думал, что уже было, но нет. И АМС Кассини отличная.
Аноним 11/09/18 Втр 11:04:50 #5 №421830 
>>421809
>Люблю раздел. Он мой родной самый
Поддерживаю, с 2014 года спейсач висит первой вкладкой, ни дня без спц.
мимо хуй, предложивший Кассини-перекат и бампающий Кассини-тредж
Аноним 11/09/18 Втр 11:09:18 #6 №421833 
>>421830
У меня вроде раньше, но суть та же.

Давайте чтобы оффтоп сильный не был спрошу тупой вопрос.

Нету ли гипотезы которая гравитацию исключает из основных взаимодействий и получается что их три и мы не нуждаемся в Теории Всего, Теории Объединения?
Гравитация же суть искажение пространства, частиц-переносчиков пока не найдено.
Аноним 11/09/18 Втр 11:10:35 #7 №421834 
кассини.jpg
Аноним 11/09/18 Втр 11:40:07 #8 №421852 
>>421833
С разморозкой, гравитация квантуется. Гугль - гравитационные волны. В 17 году нашли.
Аноним 11/09/18 Втр 11:50:49 #9 №421860 
image.png
>>421852
Где искать-то?
Аноним 11/09/18 Втр 11:51:29 #10 №421861 
>>421860
Топ кек, я себя задиванонил.
Зарепортил.
Аноним 11/09/18 Втр 12:10:57 #11 №421871 
>>421861
Тоби пизда
Аноним 11/09/18 Втр 12:18:22 #12 №421873 
image.png
>>421871
Я думаю, что ничего страшного. Но если что - я заслужил.
Аноним 11/09/18 Втр 12:38:06 #13 №421889 
Коты видят больше небесных объектов, чем люди?
Аноним 11/09/18 Втр 13:14:58 #14 №421924 
>>421889
Не больше, чем жуки навозники.
Аноним 11/09/18 Втр 13:42:45 #15 №421926 
B5099B4B-F227-4DC5-9995-5EDC80622AAE.jpeg
Есть ли какие-то положительные перспективы в обозримом будущем куда-либо долететь (где может быть жизнь)? Или это маняфантазии и никто никуда никогда не долетит?
Аноним 11/09/18 Втр 13:43:49 #16 №421927 
>>421926
На Энцеладе скорее всего есть жизнь, но лететь туда долго пиздец.
Аноним 11/09/18 Втр 13:50:54 #17 №421928 
>>421803 (OP)
https://www.youtube.com/watch?v=ShIJSpV3BII
Аноним 11/09/18 Втр 13:51:26 #18 №421929 
>>421803 (OP)
https://www.tsijournals.com/articles/the-possibility-of-ftl-space-travel-by-using-the-quantum-tunneling-effect-through-the-light-barrier.pdf
Аноним 11/09/18 Втр 13:58:11 #19 №421931 
>>421927
Последние новости: 27 июня 2018 года ученые заявили об обнаружении сложных органических макромолекул в собранных «Кассини» образцах из струйного шлейфа Энцелада.
Аноним 11/09/18 Втр 16:36:22 #20 №422006 
Промазал, бамп

>>421996
Аноним 11/09/18 Втр 16:43:22 #21 №422007 
image.png
>>422006
Картинка из англовики говорит, что кто-то пиздит. У последних экземпляров разрешение 2 фута, что порядка 0.65 метра. Ну и орбиты там довольно низкие, ниже 250км у последних.
Аноним 11/09/18 Втр 17:01:31 #22 №422012 
>>422007
Бля, перешел по первоисточникам в статье, а там вообще говорят о 15 сантиметрах на KH-11. А теоретический предел типо 7-10 сантиметров.

И если вот тут http://www.planeta-zemla.info/amkos_59.html ноунейм статья Образовательный портал о загадках Планеты Земля, УФО, НЛО, Уфология ААААААААААААААААААААА, то там еще ссылаются на

>Подполковник А.Андронов, старший лейтенант Р.Шевров «Зарубежное военное обозрение» №2 1995 г
и

>«Зарубежное военное обозрение» №3 1995 г
Аноним 11/09/18 Втр 17:10:16 #23 №422019 
image.png
>>422012
Чет не вижу я на этом пике с KH-8 объекты размером менее 10 сантиметров.
Аноним 11/09/18 Втр 17:59:12 #24 №422033 
>>421806
Ебаный стыд на месте, я спокоен
Аноним 11/09/18 Втр 18:03:53 #25 №422034 
15325050531370.jpg
Повторю тупой вопрос месячной давности: че там с фотками ЧД? Анон сказал, что какие-то диски должны в кучу свезти и начать обрабатывать данные для визуализации, или уже обрабатывают, я хуй помню. Где вообще новости по этому вопросу мониторить?
Аноним 11/09/18 Втр 18:07:30 #26 №422036 
>>422019
Понял тебя, ссу этим экспертам на лысину.
Аноним 11/09/18 Втр 18:57:22 #27 №422054 
>>422034
event horizon telescope
Аноним 12/09/18 Срд 04:27:47 #28 №422149 
400px-UZCJ224030.2+032131.jpg
Почему квазар Крест Эйнштейна так странно выглядит? Он состоит из пяти черных дыр, или как?
Аноним 12/09/18 Срд 05:03:05 #29 №422152 
>>422149
Всё, понЯл. Квазар всего один, все остальное - его отражения в галактических линзах.
Аноним 12/09/18 Срд 05:27:42 #30 №422154 
>>421927
Это всё-таки ближе планет у других звёзд.
Аноним 12/09/18 Срд 09:39:48 #31 №422178 
>>422054
Грац
Аноним 12/09/18 Срд 11:44:16 #32 №422208 
>>422034
Ее нельзя сфотографировать. Она черная в черном космосе. Такие дела.
Аноним 12/09/18 Срд 11:58:51 #33 №422211 
>>422208
Специалист дохуя? Аккреционный диск фотографируют
Аноним 12/09/18 Срд 12:04:50 #34 №422214 
>>422208
А это и не фото, это модель на основе наблюдений
Аноним 12/09/18 Срд 14:49:03 #35 №422392 
Поясните разницу между ракетой и ракетоносителем, чтобы не обосраться при знающих людях.
Аноним 12/09/18 Срд 14:54:12 #36 №422394 
>>422392
Ракетоноситель (мужской род) — средство доставки и транспортировки ракет (например, к месту пуска), «тот, кто несет ракеты».
Ракета aka Ракета-носитель (полезной нагрузки) это ракета.
В космонавтике ракетоносителей нет, ракетоносители у военов.
Ракеты-носители есть и все ракеты космического назначения (РКН) ими и являются.
Аноним 12/09/18 Срд 20:47:47 #37 №422628 
>>422394
спасибо, просто не буду употреблять слово ракетоноситель вообще.
Аноним 12/09/18 Срд 20:54:42 #38 №422635 
>>422628
В письменном виде вообще не употребляй, с большой вероятностью это неверно будет.
Когда вслух говоришь - говори в женском роде, тогда правильно звучать будет.
А то смотрел запуск Прогресса и пару раз слышал "нашего ракетоносителя" от вроде как экспертов. Сам чуть вслед за Прогрессом на орбиту не вышел.
Аноним 12/09/18 Срд 21:42:49 #39 №422671 
233b48b3ab08d197a87b8e0abdfdde86.jpg
>>422635
Хорошо, я понял. Просто тут реально есть ракета Kuaizhou 1A и у неё буквально есть ракетоноситель, и вот хрен поймешь является ли он частью ракетного комплекса называемого Kuaizhou 1A, или же ракетносителей может быть несколько разных и они не специализированы под ракету.
Аноним 12/09/18 Срд 21:55:14 #40 №422678 
nfireminotaurlaunch0 (1).jpg
563647mainIMG0490.jpg
Што это за гандон, лол? Для чего он нужен?
12/09/18 Срд 21:56:03 #41 №422679 
>>422671
Транспортно-установочные агрегаты, транспортно-пусковые платформы - тоже элементы космических ракетных комплексов. Разные КРК могут иметь общие составные части.
По названиям везде надо уточнять подробнее в каждом отдельном случае.
Аноним 12/09/18 Срд 22:00:06 #42 №422682 
SaturnSA3withraincoveronlaunchpad.jpg
>>422678
Поддерживают правильный климат внутри, труба вон снизу подведена на второй пикче.
Аноним 12/09/18 Срд 22:04:41 #43 №422686 
>>422682
Ух, первый раз вижу. Обычно набалдашник с грузом только кондиционерят.
12/09/18 Срд 22:10:19 #44 №422689 
>>422671
А на картинке CZ-6.
Аноним 12/09/18 Срд 22:12:38 #45 №422692 
>>422686
Там на первых ступенях больше по температуре вопрос, не хотят проморозить и Челленджер скосплеить.
>набалдашник с грузом
Обтекатель же.
Аноним 12/09/18 Срд 22:14:00 #46 №422693 
>>422689
Спасибо что сказал, буду внимательней.
Аноним 12/09/18 Срд 22:16:30 #47 №422694 
>>421926
Очевидно, что единственный 100% гарантированный способ обнаружить внеземную жизнь - это спалить уфошку биомаркеры в атмосферах экзопланет. Всё остальное - от лукавого. Здесь говно на покушать антисетипетуху.
Аноним 12/09/18 Срд 22:20:06 #48 №422697 
>>422694
техносигнатуры можно обнаружить еще. Если цивилизация развитая, если у них там трафик из тысяч кораблей вокруг планет, эксперименты и взрывы, майнят астероиды, светят лазерами во все стороны для ускорения и связи, строят гиганские солнечные батареи и зонтики для тени площадью под 1000 км, то таких можно будет заметить.
Аноним 12/09/18 Срд 22:49:01 #49 №422714 
Из какого металла сделан бытовой отсек корабля Союз?
Аноним 12/09/18 Срд 22:55:33 #50 №422716 
>>422714
Сплав люминя, 2мм толщина стенки + сверху ЭВТИ и противометеоритная защита.
Аноним 13/09/18 Чтв 09:27:05 #51 №422796 
>>422697
Это гораздо менее вероятно. Кислородная атмосфера существует у земляшки уже более половины ее возраста, а мегацивилизации вряд ли долго протянут. Не факт вообще, что они встречаются часто даже среди других цивилизаций.
Аноним 13/09/18 Чтв 09:36:31 #52 №422797 
На МКС есть источик искусственной гравитации или это только в фантастическом кино бывают такие ? Если есть то как он работает ? Я думал пока неизвестно откуда берётся гравитация, а потом подумал а как же её тогда на МКС делают.
Аноним 13/09/18 Чтв 09:39:41 #53 №422798 
>>422796
> Кислородная атмосфера существует у земляшки уже более половины ее возраста
Лолнет. Половину возраста земли предположительно было менее 5% свободного кислорода в атмосфере. 20-30 процентов стало относительно недавно, полмиллиарда лет назад.
Аноним 13/09/18 Чтв 09:41:20 #54 №422800 
>>422797
На МКС есть естественный источник гравитации - Земля. Сила притяжения на станции составляет 0.9g.
И все бы ничего, но МКС вместе со всем содержимым находится в свободном падении и вот уже почти 20 лет промахивается мимо Земли из-за чего все на ней испытывают эффект невесомости.
Так что нет, искусственной гравитации на МКС нет. Эта вещь есть только в кино.
Аноним 13/09/18 Чтв 11:59:59 #55 №422819 
>>422798
Изучил вопрос глубже. Внезапно, кислородным биомаркером в инфракрасном спектре поглощения является именно озон:
http://www.as.utexas.edu/astronomy/education/fall08/scalo/secure/309l_sep25_plandet.pdf
Он заметен даже при малых концентрациях двухатомного кислорода.
Аноним 13/09/18 Чтв 12:01:34 #56 №422820 
>>422819
То есть ты согласен с тем, что я сказал?

Вообще пока что озон не обнаружен в спектральных анализах экзопланет. Я бы считал его одним из важных критериев.
Аноним 13/09/18 Чтв 12:11:50 #57 №422826 
>>422820
Конечно. Я просто привлек внимание к тому, что его можно будет детектить, даже если двухатомного кислорода, условно, проценты, а не десятки процентов.
>Я бы считал его одним из важных критериев
This. Еще лучше, когда одновременно обнаруживаются озон, метан, водяной пар и углекислый газ.
Аноним 13/09/18 Чтв 12:17:14 #58 №422827 
>>422820
Вырисовывается интересная дихотомия в будущей астробиологии. Нужны будут йобы для получения спектров экзопланет и йобы для изучения спутников с океанами. Вопрос, как космические агентства потянут их финансирование, учитывая нынешнюю ситуацию с Веббом и Европа Клипером.
Аноним 13/09/18 Чтв 12:23:02 #59 №422829 
>>422826
>>422827
У тебя слегонца логическая ошибка из-за того, что ты наблюдал развитие астрономии до своих интересов скопом и теперь наблюдаешь его в реалтайме и тебе кажется, что всё замедлилось.
ИМХО вполне всё своим чередом.
Но я бы два Вебба пилил минимум, чесговоря, хорошая йоба.
Аноним 13/09/18 Чтв 12:55:58 #60 №422840 
>>422829
Нинужно. НАСА только и ждет, когда сможет пульнуть Уэбб и плотно заняться следующим космическим телескопом, уже 15-метровым.
Аноним 13/09/18 Чтв 13:02:23 #61 №422847 
amazing boner.gif
>>422840
Вообще два Уэбба лучше, чем один, и один (при одном потерянном) лучше, чем ноль. Они же щас ту же фигню с Кьюриосити проделывают, второй такой же ровер собираются на Марс пульнуть.
>уже 15-метровым
Пикрелетейтед. Люто возбудился.
Аноним 13/09/18 Чтв 13:41:43 #62 №422863 
>>422829
Ну хз, просто сложилось впечатление, что из-за мисменеджмента и недостатка финансирования (дохуя ушло на Вебба и марсоходы) пускают всякую хуимболу для аутистов.
Аноним 13/09/18 Чтв 14:14:33 #63 №422878 
>>422863
Впечатление весьма ошибочное.
Финансов космосу в идеале никогда не хватит.
Я всё лелею надежду когда он выйдет на самоокупаемость.

А прогресс идёт как шёл, сейчас отлично всё с ним, не переживай.
Аноним 13/09/18 Чтв 14:31:29 #64 №422885 
>>422878
>не переживай
Без годной фармы это невозможно.

Такто по планетной науке меня как чавкающего потребителя всё устраивает (при условии, что всё задуманное долетит и сработатет). Разве только добавить еще SAR на околовенерианскую орбиту, какие-нибудь миссии к ледяным гигантам и, пожалуй, телеком орбитер к Марсу. А вот с космическими обсерваториями тяжко.
Аноним 13/09/18 Чтв 14:46:35 #65 №422901 
>>421926
Есть пикча в большем разрешении?
Аноним 13/09/18 Чтв 14:57:46 #66 №422909 
>>422796
>мегацивилизации вряд ли долго протянут.
Ты сказал? Охуевшие совсем, не делай никаких оценочных суждений при отсутствии данных. А данных у нас нет, заметь я не упоминал чмошную хуесню под названием сфера дайсона, которую почему-то зафорсили как неизбежный шаг в развитии мегацивилизаций, а потом на основе того, что не нашли эти хуевины среди звезд стали делать какие-то выводы о том, что мегацивы невозможны. Сначала нужно было бы доказать необходимость постройки СД для мег, но этого никто не делал, просто на веру приняли.
Аноним 13/09/18 Чтв 15:15:34 #67 №422917 
Michael Griffin.jpg
>>422909
Я сказал. Тащемта у меня есть некоторые educated guesses о типичном пути эволюции развитых цивилизаций (точнее, о его космическом этапе), но анусы спейсачеров слишком узки, чтобы наполнять их этим знанием. Единственное, отмечу, что характерные временные и пространственные масштабы деятельности супер-элиенов делают их регистрацию маловероятным (если они не добрались до нашей системы).
Аноним 13/09/18 Чтв 15:17:56 #68 №422918 
>>422917
Ты общаешься не с одним спейсачером, так что посвяти в свои educated guesses и на что они основаны.
Аноним 13/09/18 Чтв 15:18:50 #69 №422920 
>>422885
Я тоже только за годные научные приборы.
Переведи 10% военного бюджета на науку и мы охуеем.
Аноним 13/09/18 Чтв 15:27:21 #70 №422923 
>>422918
>на что они основаны
>посвяти в свои educated guesses
>на что они основаны

Ну хуй знает, стоит ли. Если кратко, то на возможном развитии физики. И да, это оптимистичный прогноз.
Аноним 13/09/18 Чтв 15:30:53 #71 №422925 
>>422923
Так у тебя должны быть предпосылки. Я в них заинтересован и заинтересован в обсуждении.
Если ты считаешь что размышляешь с точки зрения научного подхода то ты должен приветствовать конструктивную критику.
А мне просто любопытно новые вещи узнать, если они окажутся новыми.
Аноним 13/09/18 Чтв 15:31:51 #72 №422926 
>>422918
А ты всерьез думаешь, что это чмо тебе что-то поведает? просто пойми, что оно черпает из такого же ограниченного колодца знаний, что и мы с тобой. Но так как любое чмо в интернете имеет гигантское ЭГО, то оно всегда считает свое мнение как EDUCATED, а всех остальных узкожопыми. Мы сейчас слишком мало данных имеем для того, чтобы с уверенностью что-то предполагать о других цивилизациях. Может оказаться, что не существует никакого типичного пути развития и цивилизации просто разные.
Аноним 13/09/18 Чтв 15:40:43 #73 №422928 
>>422926
Я не спешу переносить человека в категорию "очередное чмо с непогрешимым мнением", может быть он всё же поведает свои мысли. Если да - будет диалог. А иной раз может случиться и хороший диалог.
Если нет - то будет как ты сказал.
Аноним 13/09/18 Чтв 15:53:08 #74 №422942 
>>422928
Я предполагаю, что он просто прочитал какого-нибудь фантаста, у которого у пришельцев есть техномагия ломающая физику и решил, что раз в книге так написано, то и в жизни всё так и должно быть. Артикулировать это самостоятельно он не может, поэтому пытается в расплывчатые ответы, якобы он знает, но не хочет говорить, хотя на самом деле там и говорить нечего - просто он верит, что пришельцы умеют в магию и могут ломать законы физики. Это нельзя сказать по научному и при это не выглядеть как идиот, поэтому он предпочтет расплывчато уходить от ответов.

Моя мысль, мое чисто личное мнение состоит в том, что некоторые утечки сигналов, тепла, излучения при переходе на стадию освоения дальнего космоса скрыть никак нельзя и пришельце не будут пытаться. Пытаться скрыть все эти утечки это антитезно развитию цивилизации на этом этапе. Это как наступать на горло песне, цивилизация это как опера, но опера не может быть немой, полностью бесшумной и невидимой, немой хор это оксюморон. Но опять же я лишь предполагаю и рассуждаю в духе моего воображаемого сюжета научной фантастики, а не в области educated guesses об этом мире, не как ответ на чей-то вопрос с претензией на знание. Хуй знает какие там цивилизации могут быть, с какими принципами.
Аноним 13/09/18 Чтв 15:55:22 #75 №422945 
>>422926
>оно черпает из такого же ограниченного колодца знаний, что и мы с тобой
Ну вот я читаю некоторые разделы arxiv.org. Ограничены ли эти колодцы? Очевидно, да. Но насколько они глубоки в сравнении с безднами в башке среднестатистического брэйнлета с медиадосок?
>>422925
Одна из очевидных, но удручающе игнорируемых футурологами посылок - это эмпирический факт того, что для получения экспериментально-наблюдательной инфы для проверки естественнонаучных теорий требуются всё более и более йобистые инструменты, типа коллайдеров и телескопов. Это, ящитаю (на основании чтения текстов из arxiv.org), неслучайно и есть по сути проявление некого информационного аналога второго закона термодинамики. Только здесь вместо энтропии выступает информационная энтропия (мера неопределенности наших знаний). Собственно, глобальные перспективы зависят от того, как именно эту хрень можно будет (и будет ли?) обойти.
Аноним 13/09/18 Чтв 16:11:04 #76 №422963 
>>422945
Ты пока ничего значимого не сказал. Мы и так наблюдаем, что для определения/совладания более фундаментальных законов нужны более массивные машины.

>>422942
Ты сам немного в демагогию ударяешься. Я бы хотел придерживаться именно фактов и гипотез. Давайте говорить таким языком, не прибегая к предпосылкам.
Аноним 13/09/18 Чтв 16:14:43 #77 №422969 
>>422945
>читаю некоторые разделы arxiv.org.
Нашел чем в спейсаче выебнуться.

> эту хрень можно будет (и будет ли?) обойти
Ты даже не можешь нормально сформулировать проблему, но уже думаешь как эту хрень обойти.

Из того, что "для получения экспериментально-наблюдательной инфы для проверки естественнонаучных теорий требуются всё более и более йобистые инструменты, типа коллайдеров и телескопов" не выходит никаким образом как заключение наличие "некого информационного аналога второго закона термодинамики." Из этого только следует, что от уровня претензий теории на объяснение неких структурных уровней устройства материи требуется разные уровни энергии.

> информационная энтропия
ох, лол. Дитё еще думает, что знания являются чем-то определенным и говорят нам о мире, а не описывают его в языке. Что такое информация? Что такое знание? Как дать им определения не являющие логически круговыми? хуйня сплошная и уход от ответа, как я и говорил.
Аноним 13/09/18 Чтв 16:28:28 #78 №422985 
>>422963
Этот факт теоретически не объяснен, так как "самоочевиден". А должен был бы.
>>422969
>не выходит никаким образом
Конечно, не выходит. Но при наличии определенных знаний (которых не найдешь в твоем говносайфайчике) помимо этого факта он предстает именно в таком свете.
>на объяснение неких структурных уровней устройства материи требуется разные уровни энергии
Кек. А почему именно энергии, ты не задумывался? И да, для объяснения физики, скажем, кристаллического хрусталя и того же стекла требуются принципиально разные по сложности эксперименты и теоретические инструменты, хотя характерные энергии там одного порядка, вот э сюрпрайз!
>Дитё
>хуйня сплошная
Слив засчитан, можешь не продолжать.
Аноним 13/09/18 Чтв 16:38:07 #79 №422999 
>>422985
>Cлив засчитан,
Это ты на arxiv таким аргументам научился, дитё? Там так и представляю - спор между Харлоу Шепли и Гебером Кёртисом о размерах Вселенной и Харлоу такой "ыыы, мамку ебал", а Гебер такой "азаза слился, да? а?"
Сначала ушел от дискуссии о цивилизациях пришельцев в какую-то энтропическую болотину и неопределенную философию информации, а когда тебя обоссали за это, то начал обмазываясь говном кричать, что вы все слиты. Воистину дитё.
Аноним 13/09/18 Чтв 16:38:11 #80 №423000 
>>422985
>Этот факт теоретически не объяснен, так как "самоочевиден". А должен был бы.
Такого нет. Доказательства и подтверждения должны быть.
Будь добр предоставить, иначе твоя точка зрения просто религиозная.
Аноним 13/09/18 Чтв 16:46:10 #81 №423007 
>>422999
>рррряяяяяяяяяяя
По другим аргументам, как я вижу, комментариев нет? Так и запишем.
>>423000
Не вполне понимаю, о чем ты. Я имел в виду факт все нарастающей йобистости инструментов наблюдателя. Этот факт а) очевиден, б) теоретически не объяснен. Или, по-твоему, его не надо объяснять иначе кроме как "ну еба объяснение неких структурных уровней устройства материи требуется ептыбля разные уровни энергии хуёмое кактотак"?
Аноним 13/09/18 Чтв 16:47:42 #82 №423009 
>>423000
И да,
>религиозная
лексика напоминает таковую у антисетипетуха. Извольте объясниться, сударь!
Аноним 13/09/18 Чтв 16:50:17 #83 №423014 
>>423009
>>423007
Так не бывает таких вещей в науке как "самоочевидные". Есть постулаты в математике и всё.
Твоим словам должны быть доказательства, притом опровержимые.
Аноним 13/09/18 Чтв 16:53:21 #84 №423019 
>>423007
>Я имел в виду факт все нарастающей йобистости инструментов наблюдателя. Этот факт а) очевиден, б) теоретически не объяснен.
Не факт же.
Для наблюдения фундаментальных взаимодействий или частиц достаточно и дубнинского коллайдера. Который на минутку предсказал большинство трансурановых элементов и ДОКАЗАЛ ИХ. Несмотря на наличие БАК. Который тоже охуене и доказал Бозон Хиггса, который дубнинский коллайдер не мог.
Тем не менее хочу чтобы ты сформулировал доказательство своего "очевидного" критерия.
Наука не зиждется на "очевидности", ты должен мочь дать опровержимую и проверяемую гипотезу.
Дашь?
Аноним 13/09/18 Чтв 16:54:29 #85 №423021 
>>423014
>не бывает таких вещей в науке как "самоочевидные"
Любой наблюдаемый факт "самоочевиден".
Тем временем для продолжения дискуссии ожидаю пояснений за антисетипетуха.
Аноним 13/09/18 Чтв 16:58:31 #86 №423024 
>>423021
Зачем я должен пояснять за левого обмудка?

Ты не можешь говорить с тезисами, тебе надо говорить с личностями? Ты наступаешь на ad hominem.

Я тебе вполне валидные вопросы задал. Либо скажи, что в вопросах не так, либо ответь на них, либо окажется, что твоя позиция не так сильна с т.з. всех кто видит как ты её отстаиваешь.
Аноним 13/09/18 Чтв 17:26:11 #87 №423039 
>>423024
То есть ты не можешь сказать, "антисетипетух" ты или нет? ОК.
>как ты её отстаиваешь
To make it clear: Для меня не является самоцелью отстаивание какой-либо "позиции". Я не хожу в эти ваши интернетики с целью что-то там "популяризировать", спорить, доказывать, кого-то там "просвещать" etc. Для этого нет ни времени, ни желания.
>надо говорить с личностями
Если какая-то личность находит мои посылки sensible или хотя бы у нее есть желание и возможность самостоятельно заполнить лакуны, намеренно оставленные мной в объяснениях, то хорошо. Если нет - ОК, проходи мимо, тебя никто не держит. Срач ради срача вокруг критерия Поппера мне элементарно не интересен.
>>423019
>Дашь
Я с самого начала ее сформулировал:
>неслучайно и есть по сути проявление некоего информационного аналога второго закона термодинамики
Эта гипотеза явно фальсифицируема: если обнаружится, что скажем, квантовую теорию гравитации, когда ее создадут, можно будет подтвердить инструментом, собранным из условных палок и говна, то гипотеза жидчайше обосрется.
Аноним 13/09/18 Чтв 17:30:29 #88 №423042 
Вопрос относительно ёба-телескопов.
В чём профит бинокля перед телескопом? Важно ли расстояние между зеркалами? Есть добавить 3 зеркало будет лучше? Насколько практично запустить 2 идентичных космических телескопа, ака бинокль?
Аноним 13/09/18 Чтв 22:14:45 #89 №423112 
Насколько я понимаю, единственные пруфы океана на Европе это магнитное поле? А маньки уже на этом основываясь себе глубоководных рачков нафантазировали?
Аноним 13/09/18 Чтв 22:31:43 #90 №423118 
>>423112
Плотность, трещины и гейзеры с атмосферой.
Аноним 13/09/18 Чтв 22:31:59 #91 №423119 
>>423112
Слы, с уважением спрашивай вопрос у представителей движения подледных океанов. Ты кратеры на Европе видел? Нет, а потому что они там долго не живут. Один Пвыл и парочка других. Значится поверхность молодая. На поверхности еще есть борозды, извилины, макулы, хаосы. Чисто по чести всё говорит об океане.
Аноним 13/09/18 Чтв 22:38:47 #92 №423120 
>>423119
>Ты кратеры на Европе видел? Нет, а потому что они там долго не живут
Так туда может просто астероид побольше уебался недавно и поверхность растаяла а потом замёрзла.
Аноним 13/09/18 Чтв 22:42:20 #93 №423121 
ValhallacrateronCallisto.jpg
>>423120
Та не - если астероид побольше и недавно уебался, то его след легко был бы виден. Вот на Каллисто вьебался оче большой астероид и оставил кратер Вальхалла. А то чтобы он всю поверхность расплавил - так хуйня, без атмосферы передача тепла с одного полушария на другое невозможна, может половину расплавить, а другая бы не заметила.
Аноним 13/09/18 Чтв 23:31:05 #94 №423135 
>>423121
>может половину расплавить, а другая бы не заметила.
Скажи это миранде.
Аноним 13/09/18 Чтв 23:57:57 #95 №423139 
>>423039
Ох уж эти вставки английских слов, сразу детект псевдоинтеллектуала
>>423042
Вам в телескопотред
Аноним 14/09/18 Птн 00:34:43 #96 №423145 
>>423139
Англофоб порвался, кек.
Аноним 14/09/18 Птн 00:55:59 #97 №423152 
>>423145
Ту мэйк ит клир, лет ми спик фром май харт:
Можно, разумеется, искать англофобов даже среди тех, кто в своём посте использовал слово "детект", но я с ним согласен. Такое ничем не оправданное и случайное использование английских слов выглядит бездарно, бессмысленно и просто вырвиглазно. Тебя ещё с мелкобуквой-дислексиком из спейсикс треда скрестить, и у половины спейсача точно глаза полопаются.
Мы поняли, что ты первый в истории борды знаешь английский, все пали ниц и обоссались от счастья, теперь можешь прекращать.
Аноним 14/09/18 Птн 12:47:10 #98 №423277 
>>423039
>Если какая-то личность находит мои посылки sensible или хотя бы у нее есть желание и возможность самостоятельно заполнить лакуны, намеренно оставленные мной в объяснениях, то хорошо
Понимаешь, brother, ты тут залез в область, которая для любознательных dummies слишком сложна. Ты полез туда, где могут рискнуть разобраться only ребята с хорошим техническим багажом и взвешенными критериями. Чтобы таким became, то надо already прошерстить кучу инфы. И выработать фильтр, отбрасывающий полотнища всяких псевдонаучных shiziques. Иначе обучение будет неэффективно. Ты, будучи наглым anonimous, отсеиваешься этим фильтром сразу. Поэтому твои попытки кого-то заинтересовать подобными высокомерными empty набросами, are soilless. Заинтересуешь только совсем поверхностных залетных passing by crocodiles, которые в процессе поисков скорее прислушаются на пути к истине к Катющеку, чем докопаются до того, что ты там hide изначально.
Аноним 14/09/18 Птн 12:50:23 #99 №423279 
>>423277
Ребята, не стоит вскрывать this topic. Вы young, шутливые, вам все easy. Это не this. Это not Чикатило и even not archives спецслужб. Сюда better not лезть. Seriously, any из you will жалеть. Лучше close the topic и forget, what тут писалось. I вполне understand, that данным message вызову дополнительный interest, но хочу сразу предостеречь пытливых - stop. Остальные just не find.
Аноним 14/09/18 Птн 12:56:16 #100 №423287 
15095456942140.webm
>>423279
Аноним 14/09/18 Птн 13:06:29 #101 №423293 
>>423277
>>423279
Так эту дырявую ынтеллигенцию! Выглядит как суржик гастарбайтера
Аноним 14/09/18 Птн 13:08:19 #102 №423295 
sizes.gif
Эк вас разобрало литобрейкингом:3
Аноним 14/09/18 Птн 15:12:17 #103 №423323 
>>423295
Ожидал "Your mom". Mildly disappointed.
Аноним 14/09/18 Птн 15:15:18 #104 №423325 
15353961755900.png
Что там за инфа с обсерваторией? Неужели инопланетяне связалсь с нами!? ОТВЕТЕ!
Аноним 14/09/18 Птн 15:16:01 #105 №423326 
>>423325
Нет.
Аноним 14/09/18 Птн 15:35:17 #106 №423332 
>>423325
Не. Речь о вот этих штуках:

https://www.youtube.com/watch?v=arIOVRSOLsg
https://www.youtube.com/watch?v=k8ediKuYyKI

Второе видео покороче, первое -- длинная лекция на данную тему.
Аноним 14/09/18 Птн 15:38:50 #107 №423334 
>>423332
Солнечная обсерватория и FRB? Или вы вообще о чём блядь?
Аноним 14/09/18 Птн 15:44:33 #108 №423335 
>>423334
То есть вопрос был не про вот это?

https://techcrunch.com/2018/09/10/seti-neural-networks-spot-dozens-of-new-mysterious-signals-emanating-from-distant-galaxy/
Аноним 14/09/18 Птн 15:48:37 #109 №423336 
>>423335
А хуй знает. Там вон еще https://nplus1.ru/news/2018/09/13/solar-observatory-closed , я на это сперва наткнулся в попытках понять о чём речь вообще.
Аноним 14/09/18 Птн 17:30:57 #110 №423366 
>>423323
>Ожидал
Лол, гипотеза не получила подтверждения!
Аноним 14/09/18 Птн 20:00:38 #111 №423420 
>>421803 (OP)
Проектная работа
Сап. Хочу запилить ракету(миниатюрную естественна) собственноручно. Вроде какие-то гайды видел, вроде не слишком сложно.
Проконсультироваться не с кем, пишу сюда. Что скажете? Муторно ли? Если учесть, что мне придется несколько раз представлять эту работу.
Аноним 14/09/18 Птн 23:04:58 #112 №423470 
>>423420
Лучше в этот тред обратись: https://2ch.hk/spc/res/410304.html

Твердотопливную карамельную ракету сделать несложно и не особо муторно, но она, естественно, одноразовая. Жидкостную с движками, способными включаться несколько раз, в одиночку ты не соберешь.
Аноним 14/09/18 Птн 23:21:30 #113 №423478 
1486840862855.jpg
Поясните почему никто не хочет пиарить идею полета к Фобосу и Деймосу. Это же очевидно, что мимо них не пройти, что первый пилотируемый полёт к Марсу будет без высадки на поверхность, потому что на Луну тоже сел не Аполлон-1 или 2, а Аполлон -11. Следовательно нужно будет прилететь к Марсу, облететь вокруг него и вернуться без посадки и делая это можно как минимум сесть на Фобос. Если пропиарить эту тему, поднять хайп, то первая высадка на Фобос была бы такой же по значимости как высадка на Луну и Марс, всё таки это уже дальний космос, еще не Марс, но уже не Луна. Но никто не говорит о Фобосе, как воды в рот набрали как будто он не существует. Реально что-ли суеверничают из-за названия и боятся, что миссию к Фобосу постигнет фейл?
Аноним 14/09/18 Птн 23:23:15 #114 №423482 
>>423478
Фобос - говно без задач. Туда достаточно послать АМС. Даже необязательно покидать землю притом, можно в грунт уронить, настолько фобос уныл.
Аноним 14/09/18 Птн 23:32:14 #115 №423488 
1047px-221831mainPIA10368.png
>>423482
Фобос возможно образовался из грунта Марса когда-то - если взять с него пробы(и я имею ввиду нормальные пробы, а не 100 грамм пыли с 2 мм поверхности в глубину), то считай можно не садясь на Марс получить хоть какой-то марсогрунт в нормальной дозе, не поднимая его со дна колодца. Плюс тренировка для нормальных миссий к астероидам и добычи на них, далеко за пределами орбиты Земли, там где как раз дохуя астероидов. Можно отработать настоящие технологии. Можно кругосветное путешествие вокруг Фобоса сделать. Нормально померить его недра, собрать дерьмо со дна его кратеров. В реальности вам не КСП, где на астероид 1 или 4 биома, там в каждом кратере можно что-то интересное найти.
Аноним 15/09/18 Суб 07:55:52 #116 №423529 
>>423478
Выход на орбиту Марса и посадка на Фобос будут энергозатратнее, чем посадка на Марс. Потому что на Марс садятся с разбегу, прямиком с Земли, поскольку скорость гасится за счет торможения об атмосферу.
Аноним 15/09/18 Суб 11:33:06 #117 №423552 
>>423478
Пропиарить посадку людей на кусок космического говна невозможно. При абезьянке пытались пиарить захват мелкого околоземного астероида или булыжника с астероида, с последующим изучением огурцами на Орионе. Всем было похуй, и от проекта отказались в пользу лунной ОПГ. А про другие Фобосы ты и сам знаешь. Вот очередной:
https://en.m.wikipedia.org/wiki/Martian_Moons_Exploration
Аноним 15/09/18 Суб 11:49:06 #118 №423554 
>>423529
Схуяли посадка энергозатратней? На фобос можно точно так же аэродинамически тормозить (хотя вообще-то для большого аппарата это сомнительная затея). Там на одну реактивную посадку пиздец сколько уйдёт. (а другой не получится)

>>423478
>Поясните почему никто не хочет пиарить идею полета к Фобосу и Деймосу.
Вообще-то в определенный момент у насы была идея промежуточного полёта к Фобосу в пилотируемой марсианской программе, или даже станции там.
Аноним 15/09/18 Суб 11:55:09 #119 №423556 
>>423554
>Схуяли фобос энергозатратней?
фикс
Аноним 15/09/18 Суб 12:01:28 #120 №423559 
>>423552
Обезьянка думал, что там дохуя астероидов, к которым Орион может лететь, а там оказалось что их-то дохуя, но не дохуя чтобы долететь в срок и возможности корабля и ракет ограничивают выборку, плюс можно было поймать и слетать лишь к мелкому, а космические геологи говорили, что мелкого говна мы и так в Антарктиде насобирали, не надо нам такого счастья "мы хондритов что-ли невидали? не приплетайте нас к этой авантюре".
А Фобос уникален - для примера сравни эту пикчу>>423488 и Рюгу. На Фобосе почти нет камней, которые на Рюгу как слепни сидят. Но при этом на Фобосе есть своя аномалия, свой монолит
https://en.wikipedia.org/wiki/Phobos_monolith
Аноним 15/09/18 Суб 12:03:43 #121 №423560 
1473629947343.jpg
>>423552
>Martian_Moons_Exploration
>The lander mission aims to retrieve a minimum 10 g (0.35 oz) of samples.
>10 g
Аноним 15/09/18 Суб 12:36:08 #122 №423569 
>>423559
Фобос уникален лишь тем, что крутится вокруг Марса, а не других объектов, и, как следствие, несет следы "совместного проживания" в виде борозд и отложений из импактных выбросов.
Аноним 15/09/18 Суб 13:03:56 #123 №423570 
>>423470
Cпасибо)
Аноним 15/09/18 Суб 15:22:43 #124 №423631 
https://naked-science.ru/article/sci/gravitacionnye-volny-ukazali-na
Значит, все манятеории с одиннадцатью измерениями всё?
Аноним 15/09/18 Суб 16:07:09 #125 №423636 
>>423631
Нет, это значит лишь, что не существует макроскопических дополнительных измерений. Микроскопические компактные пространства, в которых волны не помещаются, никак не опровергаются этими манянаблюдениями.
Аноним 15/09/18 Суб 16:25:01 #126 №423639 
>>423559
Так камни сползли просто, вон какие борозды по всему Фобосу прочерчены. Внутри он не монолит, а точно такая же куча отдельных камней с пустотами между ними, как и Рюгу.
Аноним 16/09/18 Вск 08:30:01 #127 №423851 
>Если бы только мыть. Но периодически надо менять элементы этой сантехники. А иногда чтото ломается, типа насосов-сепараторов и их приходится менять с извлечением всего что они не рассепарировали.
>Делают всё это естественно сами Герои Космоса надев маски (чтоб не плеснуло в морду) и перчатки.
>Так что реализация вековой мечты человечества выглядит вблизи очень неромантично. Причём освоятели космоса своими руками всё это прекрасно знают и долго к этому готовятся, тренируются. Кому не нравится - ничего не мешает написать рапорт и перевалифицироваться в управдомы.

>Насадки только под пи-пи. С ка-ка хуже. Всё делается в один унитаз и каждый убирает за собой.
>Причём как живописно рассказывала Сунита Уильямс иногда это дело разлетается и его приходится собирать салфетками из воздуха и со стен.

Пипец, реальне в космосе негигиенично срать, всё разлетается? Я думал там наоборот, всё буквально "высасывает", чистенько, хуе муе...
Аноним 16/09/18 Вск 13:59:58 #128 №423883 
>>423851
>реальне в космосе негигиенично срать, всё разлетается
Не только. Оно еще и облепляет все, т.к. отлично работают силы поверхностного натяжения.
Аноним 16/09/18 Вск 14:07:49 #129 №423886 
>>423851
>>423883
Да, а почему нельзя присосать к жопе раструб, который будет засасывать говно прямо из жопы?
Аноним 16/09/18 Вск 14:09:17 #130 №423890 
>>423886
На стенках трубы говны собираться будут, забиваться, чистить все равно надо, в тоге при разборке говны разлетятся
Аноним 16/09/18 Вск 14:44:28 #131 №423893 
Есть геофизическая ракета, которая улетает на высоту 2500 км. Какая у неё будет вертикальная скорость при вхождении в атмосферу Земли на высоте в 100 км?

Около 7 км/с? Но это слишком много.
Аноним 16/09/18 Вск 14:53:40 #132 №423895 
>>423893
>Какая у неё будет вертикальная скорость при вхождении в атмосферу Земли на высоте в 100 км
v = sqrt(2*g(2500-100)) = 6862 м/с
Скорость будет меньше вычисленной, т.к. g - функция от высоты, но сойдет
Аноним 16/09/18 Вск 15:00:45 #133 №423897 
image.png
>>423893
Сила притяжения на 2500км от земли составляет 5,1 м/с2
Возьмем среднее ускорение за 7 м/с2 (он дольше в отдалении находится, не хочу ебаться с расчетами).
V2 = 2 x a x d
d = 2400 km, a = 7 m/s2
v = sqrt (2 x 2400 000 m x 7 m/s2) = 5 786,55

Примерно 5,800 м/сек.

>>423895
Же сильно падает, пикрил притяжение на высотах 2500, 1200 и 0 соответственно.
Аноним 16/09/18 Вск 15:04:44 #134 №423900 
>>423895
Но ведь получается для такого полета ей надо иметь ту же самую скорость в конце разгона. Это же почти орбитальная ракета нужна. С учетом потерь грубо получится 7.5-8 км/с характеристической скорости нужно. При 8.5-9 км/с для выхода на орбиту.

Тут точно нету принципиальной ошибки?

Хуево, я с посанами хотел нормальный космический туризм в несколько часов полета в невесомости и фотографиями Земли с высоты геостационарных спутников сделать. Получается очень сложно и не нужно.
Аноним 16/09/18 Вск 15:08:49 #135 №423901 
>>423900
У метеоракет потери мизерные относительно орбитальных, это раз: им не надо толщи атмосферы лежа пронзать, поддерживая свою массу, удаляясь чисто вверх ты очень быстро проходишь атмосферу и макс-кью, так что можно ускоряться быстрее, чем орбитальные ракеты, что в свою очередь уменьшает и гравитационные потери.

И да, вот цифра более верная: >>423897
>Примерно 5,800 м/сек

А теперь вспомни закон Циолковского и пойми, что еще одна сотенка дельты - это тонны топлива, еще одна - это более йобистая ракета, там экспонента стоит ведь.
Аноним 16/09/18 Вск 15:10:21 #136 №423902 
>>423900
Не, эта скорость достигается за счет ускорения свободного падения с высоты 2500км.
Аноним 16/09/18 Вск 15:13:31 #137 №423904 
>>423902
Что "не"? Абсолютно та же дельта тратится и на достижение этой высоты.
Аноним 16/09/18 Вск 15:23:35 #138 №423907 
>>423900
>с высоты геостационарных спутников сделать
Че блять?.шебм
Аноним 16/09/18 Вск 15:25:03 #139 №423908 
>>423907
Что не так? Технически это возможно, только мелкоракету не спасешь, сгорит в атмосфере к хуям.
Аноним 16/09/18 Вск 15:26:34 #140 №423910 
>>423908
ГСО это 35к километров
Аноним 16/09/18 Вск 15:27:16 #141 №423911 
>>423910
И? По-твоему суборбитальная траектория до/выше 35 тысяч километров невозможна?
Аноним 16/09/18 Вск 15:29:00 #142 №423912 
>>423911
Это сложно, еблан.
Вон Безос банку свою едва выше сотки пускать собрался
Аноним 16/09/18 Вск 15:29:56 #143 №423914 
>>423912
Никто не говорил, что это просто, еблан.
Кроме анона задавшего тупой вопрос.
Аноним 16/09/18 Вск 15:34:19 #144 №423917 
>>423900
>точно нету принципиальной ошибки
А че не так то ? Типа хотел наебать физику, тем, что не делать разгон вбок ? Если чуть подумаешь, то поймешь, что суть тут в гравиполе, которое работает одинаково пиздато во все стороны.
Аноним 16/09/18 Вск 16:11:39 #145 №423923 
>>423911
На практике нет, из-за гравипотерь при ускорении вверх.
Аноним 16/09/18 Вск 16:24:16 #146 №423924 
14626310866870.jpg
Что отвечают сторонники космической колонизации на вопрос "почему мы не колонизируем Антарктику?"?
Аноним 16/09/18 Вск 16:31:40 #147 №423925 
>>423924
Реалисты о колонизации речи не заводят. Энтузиасты хотят флаговтык и базу на Луне или экспедицию на Марс. А мечтатели ничего не отвечают, у них нету рационального зерна. Просто хотят.
Аноним 16/09/18 Вск 16:31:46 #148 №423926 
>>423924 ничего
Аноним 16/09/18 Вск 16:35:44 #149 №423927 
>>423923
Обоснуй.
Но сразу скажу: хуйню несёшь.
Аноним 16/09/18 Вск 16:39:54 #150 №423928 
image.png
P2020027.jpg
image.png
>>423924
Так мы уже.
Только это принципиально колонизацией не является, т.к. у колонизации цель - дать возможность человечеству продолжить существовать, если земляшке станет пиздец например от случайного астероида-убийцы.
Самоподдерживающаяся база на антарктике человечеству не поможет, ее может распидорасить с остальным на поверхности. На Марсе или даже на Луне - может помочь.

Вот тебе такой ответец.
Аноним 16/09/18 Вск 16:45:25 #151 №423930 
>>423928
500 человек на полюсе - это не колонизация
Аноним 16/09/18 Вск 16:46:28 #152 №423931 
>>423930
>Только это принципиально колонизацией не является
Ты чем читаешь, жуеба?
Аноним 16/09/18 Вск 16:47:45 #153 №423932 
>>423927
Каждая секунда полета вверх стоит 9,8 м/с дельты. При движении вбок этого нет.
Аноним 16/09/18 Вск 16:48:53 #154 №423933 
fukken lold.png
>>423932
ЕБАААТЬ ПИЗДЕЦ.
Аноним 16/09/18 Вск 16:50:35 #155 №423934 
>>423933
Хули ты ржешь пидор?
Аноним 16/09/18 Вск 16:59:11 #156 №423935 
>>423934
Давно такую смешную хуйню не читал.
Как и предсказывал, несёшь хуйню.
Ты либо сравниваешь полёт от земли к ГСО напрямую с перелётом от орбиты на ГСО, либо принципиально не понимаешь сути гравитационных потерь.
Аноним 16/09/18 Вск 17:15:59 #157 №423937 
>>423931
жуебами
Аноним 16/09/18 Вск 17:49:14 #158 №423942 
>>423935
Я сравниваю постоянное ускорение строго вверх до тех пор пока апогей не станет 35000км с обычным выходом на орбиту.
Аноним 16/09/18 Вск 17:55:52 #159 №423944 
>>423942
Но зачем? И каким образом это мешает долететь до высоты 36ккм летя строго вверх?
Аноним 16/09/18 Вск 17:57:08 #160 №423946 
Почему во время дождя пахнет озоном, даже если грозы нет?
Аноним 16/09/18 Вск 18:00:04 #161 №423948 
>>423946
https://ru.wikipedia.org/wiki/Петрикор
Было проведено около 600 экспериментов на 28 различных поверхностях, в том числе инженерных материалах и образцах почвы. Когда капля попадает на пористую поверхность, воздух из её пор формирует пузырьки, которые, в свою очередь, производят аэрозоль[5]. Такие аэрозоли переносят аромат, а также бактерии и вирусы из грунта. Капли дождя, которые движутся с меньшей скоростью, как правило, производят больше аэрозолей — это служит объяснением того, почему петрикор чаще появляется после лёгкого дождя
Аноним 16/09/18 Вск 18:18:21 #162 №423955 
>>423944
Пока ты наберешь апогей 35 тыс км у тебя столько дельты проебется от гравипотерь, что может даже с выходом на орбиту проще будет.
Аноним 16/09/18 Вск 18:39:28 #163 №423957 
>>423932
Бляяя, всё-таки насрал...
Аноним 16/09/18 Вск 18:56:31 #164 №423959 
>>423928
>у колонизации цель - дать возможность человечеству продолжить существовать, если земляшке станет пиздец например от случайного астероида-убийцы.
Что это за астероид такой, после которого на Земле меньше людей останется, чем в колонии и как часто астроиды такого размера на Землю падают? Что мешает эту колонию здесь же на Земле в подземном бункере сделать?
Аноним 16/09/18 Вск 19:00:03 #165 №423960 
>>423959
>Что это за астероид такой, после которого на Земле меньше людей останется, чем в колонии
Это такой астероид, который проломит тектонические плиты и всем пизда.
>и как часто астроиды такого размера на Землю падают?
Нечасто, но нам и одного раза хватит.
Аноним 16/09/18 Вск 19:54:01 #166 №423969 
>>423960
>Нечасто
"Нечасто" это что? Когда в последний раз такой астероид прилетал?
Аноним 16/09/18 Вск 22:09:27 #167 №424004 
Двач. Я решил проблему радиации в дальних перелетах. Окружаем отсек с космонавтами бутылкообразным баком с горючкой. Бак с горючкой - окружаем бутылкообразным баком с окислителем. "Горлышко" - это вход/выход для людей. Придаем кораблю осевое вращение - горючка и окислитель равномерно распределяются по стенкам баков центробежной силой, поглощая часть радиации, не являясь бесполезной массой.
Аноним 16/09/18 Вск 22:13:21 #168 №424009 
>>424004
Есть решение проще - проходишь пояса Ван Аллена через высокое наклонение с большой скоростью, а потом просто ориентируешь корабль жопой к солнцу, носом с огурцами вперед, в итоге закрываешься ВСЕМ КОРАБЛЬЕМ от солнечных вспышек и говна от него (а это единственный значимый источник радиации в ближайшей солнечной системе кроме радиационных поясов)
Аноним 16/09/18 Вск 22:15:59 #169 №424015 
>>424009
Спасибо
Аноним 16/09/18 Вск 22:32:39 #170 №424023 
>>424015
С тебя бутылка хорошего южноафриканского вина
Аноним 16/09/18 Вск 22:42:18 #171 №424024 
>>422800
>Сила притяжения на станции составляет 0.9g.
Чет дохуя, это откуда?
Аноним 16/09/18 Вск 22:42:58 #172 №424025 
Сап, нужно намутить спутниковый снимок определенной территории в мск, для фотограмметрии. какой ресурс моржно заюзать? чем качественнеее тем лучше
Аноним 16/09/18 Вск 22:51:33 #173 №424026 
ISS gravity.png
>>424024
https://en.wikipedia.org/wiki/Surface_gravity#Mass,_radius_and_surface_gravity

А дофига потому что высота всего 400 км.
Аноним 16/09/18 Вск 23:01:46 #174 №424031 
>>424026
Так, подожди. Какая разница, что там 88% от гравитации на поверхности, если консервная банка в свободном падении?
Аноним 16/09/18 Вск 23:12:33 #175 №424035 
>>424031
На практике никакой разницы, конечно. Ну за исключением каких-нибудь приливных сил, но МКС на них наплевать. (Не наплевать было бы станции Freedom, на которой планировалось их использовать для "бесплатного" поддержания ориентации.)
Аноним 16/09/18 Вск 23:16:55 #176 №424038 
>>424035
Просто тот анон написал про 0.9g, а падений на МКС даже в десять раз медленнее чем на Земле я как-то не наблюдал
Аноним 16/09/18 Вск 23:19:20 #177 №424039 
>>424038
Просто ты тупой. МКС вся находится в свободном падении
Аноним 16/09/18 Вск 23:19:36 #178 №424040 
>>424038
Ну так сила тяжести никуда не девается.
Аноним 16/09/18 Вск 23:30:13 #179 №424051 
>>424039
Сам ты тупой. МКС вся находится в свободном падении
Аноним 16/09/18 Вск 23:34:37 #180 №424056 
Могут ли звёзды находиться вне галактик?
Аноним 16/09/18 Вск 23:35:47 #181 №424057 
>>424056
А что им по-твоему должно помешать?
Аноним 16/09/18 Вск 23:40:20 #182 №424061 
vcstarsillustbig.jpg
>>424056
Да. Более того, такие звёзды известны.

http://hubblesite.org/news_release/news/1997-02
https://apod.nasa.gov/apod/ap020119.html
https://en.wikipedia.org/wiki/Intergalactic_star

>intergalactic stars are now generally thought to have originated in galaxies, like other stars, but later expelled
Аноним 16/09/18 Вск 23:55:08 #183 №424070 
>>424061
Благодарю. Блджад, даже на Вики статья есть, я мог бы и погуглить.
Аноним 17/09/18 Пнд 03:28:46 #184 №424085 
Нашел пост в /б/.

Кто-нить может аргументированно по нему ответить?


Лахта служит своим царям и пытается утопить все треды о лунной афере сша в своем дерьме. Вы спорите о мелочах, но давайте спросим лахту и их госдеповских кураторов о главном - где результаты тех. достижений полетов на луну? Где научные результаты при многократный полетах на Луну? О чем идет речь? дело в том что любой техпрогресс развивается после окончания спецпрограммы. Например - интернет изначально был программой спецсвязи на случай войны, и только потом он стал развиваться как средство связи всего мира. Жестяная банка для транспортировки жратвы из СШАШ в Европу(в отсутствии холодильников) стала обыденной консервой.
В чем пиздецома и смех лунного наебалова сша? В отсутствии развития технологии. Вдумайтесь только, согласно легенде НАСА было создано три ракетных носителя Сатурн - Сатурн 1, Сатурн 5, и гибрид между ними Сатурн 1б. Эти носители выводили фантастическую нагрузку не куда-нибудь - на Луну! в 60-70-х годах 20-го века! И потом вдруг янки убирают все эти Сатурны(они исчезают полностью, в музее НАСА есть лишь картонный макет Сатурна), перехоят на голимейшую программу Спейс-Шаттл, терпят массу крушений, и в конце-концов переходят на старые советские Протоны, которые летают безотказно до сих пор. Как это понять? Поясню ещё раз. Есть реактивный двигатель на Ме 242. И есть люди, которые, поняв идею, его развивают - через 10 лет после окончания ВОВ все ведущие страны мира имеют самолеты с реактивными движками. А тут США забрасывают на Луну фантастический, по меркам даже нынешнего времени груз - и абсолютно никакого развития темы. ну не пиздец ли? Пидорнутая лахта, что ты скажешь мне в ответ
sage[mailto:sage] Аноним 17/09/18 Пнд 03:31:35 #185 №424086 
>>424085
Уноси обратно, нет не можем
Аноним 17/09/18 Пнд 03:33:50 #186 №424087 
>>424056
В общем-то и планеты вполне могут не принадлежать звёздным системам, а просто летать сами по себе. Более того, существует предположение, что таких планет больше чем тех что крутятся вокруг звёзд.
Аноним 17/09/18 Пнд 03:41:46 #187 №424090 
>>424086
Конечно не можешь, ты же дегенерат.

Мне кажется, что и на сегодняшний день еще нет технологий позволяющих с Луны вернуть на землю человека. Посмотрите на Байконур, или на мыс Канаверал, откуда стартуют корабли с людьми, какие там коммуникации и оборудование. На Луне этого ничего нет, да и прилунить достаточное количество топлива в спускаемом модуле без детонации, тоже весьма проблематично. А о том, как в том спускаемом аппарате (на картинке из лунной экспедиции) уместилось достаточное для обратной дороги количество топлива - большой секрет, к тому же там даже видимости двигателей нет. По мне, так это вообще клоунада, для деньгодателей. Или 6 аргументов, почему полет американцев на Луну был невозможен 1. Любая кино- или фотопленка (цифровой аппаратуры в то время еще не существовало), на которую нанесен светочувствительный эмульсионный слой, чувствительна не только к свету, но и к невидимому радиационному излучению, в том числе и космическому. Рентгеновское излучение вне магнитного поля Земли превышает допустимое для человека в сотни раз! На Луне фотоаппараты и кинокамеры должны были, так же как и астронавты, быть надежно защищены от губительной космической радиации. Например, прятать их в толстые свинцовые боксы. Но кто ж будет делать космический корабль из свинца! Что мы видим на снимках? Камера жестко закреплена на груди астронавта, при этом у нее вообще нет никакой защиты. Если бы прилунение «Аполлона-11» действительно состоялось, то астронавты привезли бы на Землю кассеты с засвеченной пленкой. А еще посмотрите на перчатки астронавтов: разве могли они, не снимая их, установить выдержку и резкость? Да еще и не глядя в видоискатель. А ведь снимки получились потрясающие по композиции и цвету! 2. Фотографии, якобы сделанные на Луне, представлены НАСА с обрезаниями и исправлениями: в некоторых местах удалены тени, наложена ретушь. На некоторых снимках четко видны тени от двух, а то и трех источников света, что на Луне невозможно. Там источник света один - Солнце. Одни и те же снимки, которые НАСА предоставило в разное время, выглядят по-разному и неопровержимо доказывают наличие монтажа. На подавляющем большинстве снимков не видно звезд, просто черный пустой фон, хотя на снимках, которые сделаны с советских космических кораблей, их изобилие. Это можно объяснить лишь тем, что в павильоне трудно с точностью смоделировать звездное небо. 3. Лунный пейзаж на снимках НАСА удивительно скучный и напоминает обычный песчаный пляж, щедро посыпанный цементной пылью. Это совершено не вяжется с известными нам снимками лунной поверхности, которая завораживает обилием кратеров разной величины. Зачем же нужен был влажный песок и цемент? Очень просто: вспомните знаменитый снимок, который обошел все страницы мировой прессы - след первого человека на Луне! Ему аплодировал весь мир! Разве возможно сделать четкий ребристый отпечаток человеческого следа на реголите (лунной пыли)? А вот на цементе - запросто. И еще интересное наблюдение: реактивная струя, бьющая из сопла спускаемого на Луну аппарата, должна была разметать в условиях малой силы тяжести всю пыль - практически невесомую - с поверхности в радиусе минимум сотни метров. В безвоздушном пространстве эта пыль должна подняться высоко над поверхностью Луны и улететь на километры от места спуска корабля. Именно такая ситуация была при посадках всех советских лунных модулей. Однако на американских снимках - вопреки науке и здравому смыслу - мы видим, как только прилетевший астронавт бодро прыгает с прилунившегося аппарата в нетронутую пыль и топчется под самым соплом, оставляя повсюду свои исторические следы. Посмотрите, лунной пыли нет и на обшивке самого посадочном модуля. Все чисто и стерильно! Да и сам его внешний вид удивляет - он обклеен золотистой фольгой, причем в некоторых местах видны следы скотча. Как тут не улыбнуться и не вспомнить удивительный летающий аппарат из фильма Георгия Данелия «Кин-дза-дза»! 4. В момент установки флага США на Луне он колыхался под воздействием воздушных потоков. Но мы знаем, что на Луне нет атмосферы. Армстронг поправил флаг и сделал несколько шагов назад. Однако флаг не перестал колыхаться. Никакими «внутренними колебаниями флага» или его «внутренней энергией» это объяснить нельзя. 5. Скафандры «Аполло» 60-х годов значительно меньше советских и американских скафандров, использующихся сегодня для выхода в космос на непродолжительное время. Астронавты ходили по Луне в резиново-матерчатых скафандрах, которые были почти на сотню килограммов легче, чем тот же сконструированный в СССР свинцовый лунный скафандр Леонова. Даже при сегодняшнем уровне развития технологий в такие скафандры невозможно уместить запас кислорода на 4 часа, радиостанцию, систему жизнеобеспечения, систему терморегулирования и прочее. Заметьте, астронавты всех лунных миссий «Аполлон» вернулись на Землю живыми и здоровыми, никто из них впоследствии не умер и не пострадал от лучевой болезни. 6. Видеоролики, представленные НАСА как неоспоримое доказательство высадки астронавтов на Луну, также вызывают сомнения в подлинности. Если «лунные» фотографии были высочайшего технического качества, то видеоматериалы, выложенные в интернет, настолько плохого качества, что проводить по ним профессиональную экспертизу - дело чрезвычайно сложное. Я думаю, таким образом пытаются скрыть от зрителя всю декорационно-постановочно-каскадерскую фальшь. Чтобы изобразить легкость, с которой астронавты передвигались по Луне, их подвешивали на струны. Впоследствии струны удаляли при помощи так называемого метода блуждающей маски. Этот прием комбинированных съемок очень часто использовался в мировом кинематографе в 70 - 80 годы, до появления технологии компьютерной обработки. Ну а сегодня удалить металлическую струну в цифровом изображении не составляет особого труда. С этой задачей может справиться даже начинающий компьютерный дизайнер. Возможно, многие видеоролики с астронавтами на Луне исправлялись подобным образом уже в наши дни.
Аноним 17/09/18 Пнд 03:46:29 #188 №424092 
>>424025
Фотограмметрия намного лучше идёт с аэрофотосъёмкой. И не снимок, а снимки, тебе же с дохуя сторон снять надо, для фотограмметрии-то? Может тебе просто готовая карта высот пойдёт, с открытого SAR датасета? Разрешение/охват/оперативность какие нужны? Какого размера объекты надо перевести в 3D?
>чем качественнеее тем лучше
Это понятие очень размытое.
Аноним 17/09/18 Пнд 04:42:17 #189 №424100 
>>424090
Охуенная паста, давно так не проигрывал.
>Мне кажется
Дык креститься надо
>Посмотрите на Байконур, или на мыс Канаверал, откуда стартуют корабли с людьми, какие там коммуникации и оборудование. На Луне этого ничего нет
Посмотрите на автозаправку, какие там коммуникации, на парковке возле ашана ничего этого нет.
>прилунить достаточное количество топлива в спускаемом модуле без детонации, тоже весьма проблематично.
Ведь аэразин-50, как известно, детонирует даже от пердежа за стенкой на соседнем заводе, а LM садился на 150 м/с.
>Рентгеновское излучение вне магнитного поля Земли превышает допустимое для человека в сотни раз!
Да, одним рентгеном пробивает рельсу вдоль, а пояса Ван Аллена аж до Плутона тянутся!
И вот в таком духе вся остальная паста, я аж прослезился и схоронил.
Аноним 17/09/18 Пнд 08:36:03 #190 №424105 
>>424070
Не переживай. Ты молодец, что задал тупой интересный вопрос, ради тебя и подобных тебе тред и есть.
Аноним 17/09/18 Пнд 11:31:20 #191 №424115 
>>424100
А ты любопытный. Я дальше картонного макета "Сатурна" не читал.
Аноним 17/09/18 Пнд 11:37:53 #192 №424117 
Зачем марс, если есть титан и европа с океанами?
Аноним 17/09/18 Пнд 11:44:50 #193 №424119 
>>424117
Ты туда лететь ебанешься.
Аноним 17/09/18 Пнд 11:55:06 #194 №424121 
анончики, поясните за каналы на марсе, плз. были каналы, куча народу их видела, рисовала. потом так хоп. они пропали. послали станцию - станция их не увидела. ну ок, значит обман зрения, иллюзия. вопрос: какого хрена эту иллюзию больше никто не видел с тех пор? например мираж - иллюзия, но он от этого не перестал появляться.
Аноним 17/09/18 Пнд 12:05:08 #195 №424122 
>>424121
Потому что после фотачек никому больше в хуй не вперлось часами аутировать на мыльное рыжее пятнышко в объективе.

>мираж
Мираж - оптическое явление, существующее независимо от глаза лысой обезьяны. Его можно сфоткать. А планетарная сеть каналов существовала только в воспаленных мозгах свидетелей марсианской разумной жизни. Те "каналы", которые удавалось сфотографировать, оказались линейными структурами вроде долин Маринера.
Аноним 17/09/18 Пнд 12:19:58 #196 №424124 
>>424122
ну ладно, не мираж. иллюзия. обман зрения. можно про эти каналы где-то подробно почитать? я в инете почему-то ничего не нашел где было бы конкретно объяснено как пришли к тому что это обман зрения
Аноним 17/09/18 Пнд 12:27:51 #197 №424125 
>>424124
А поцчему ви спrашиваете? Нейросайентист дохуя?
Аноним 17/09/18 Пнд 12:30:59 #198 №424126 
>>424125
ну прост не верю и все. хз. если бы хоть какие-то каналы было видно, были бы фотки, в том числе современные. сейчас у каждого аутиста есть телескоп
Аноним 17/09/18 Пнд 14:28:25 #199 №424138 
>>424126
https://www.skyandtelescope.com/astronomy-news/venus-spokes-an-explanation-at-last/?c=y
Аноним 17/09/18 Пнд 14:35:13 #200 №424141 
>>424126
> сейчас у каждого аутиста есть телескоп
Сейчас каждый аутист знает, что нет никаких каналов, потому и не видит. Когда-то кто-то что-то увидел, и остальные тоже начали находить, потому что им очень хотелось, чтобы там что-то было, вот и искали каналы в том, что, на самом деле, было плохом воображения уже а не тем, что реально видели. Не зря фото каналов никто не делал, только зарисовки, хотя во времена того же Лоувелла фотография в астронимии применялась уже.
Аноним 17/09/18 Пнд 20:27:15 #201 №424229 
>>424121
Из-за восприятия глаз и примитивной оптики.
Щас купи телескоп-рефрактор за 10к и пялься на марс - будешь видеть эдакие особенности
Аноним 18/09/18 Втр 00:19:07 #202 №424256 
>>424117
Гравитационный колодец Юпитера. И лететь охуеть сколько.
Аноним 18/09/18 Втр 00:46:50 #203 №424264 
>>424256
Какая высота геостационарной орбиты обратной стороны Луны ?
Аноним 18/09/18 Втр 00:47:41 #204 №424266 
>>424264
Ненаправленный вопрос,миссклик.
самофикс
Аноним 18/09/18 Втр 00:52:15 #205 №424267 
>>424264
Вторая ссылка в гугле

https://www.quora.com/Does-Moon-have-geostationary-orbit
https://www.quora.com/Does-the-Moon-have-a-geostationary-orbit-If-so-what-is-its-altitude
Аноним 18/09/18 Втр 00:53:48 #206 №424268 
>>424264
>геостационарной
>гео
Znacheniye znayesh?
Аноним 18/09/18 Втр 00:55:22 #207 №424269 
>>424267
67к километров,спс,просто доверился яндексу.
>>424268
селена стационарная,пойдёт ?
Аноним 18/09/18 Втр 01:05:32 #208 №424273 
>>424269
>67к километров
Блэт, чем ты читаешь, стационарная орбита вокруг Луны это где-то 88000 км, на 66 заканчивается сфера Хилла, из-за которой стационар у Луны и невозможен.
Аноним 18/09/18 Втр 01:07:46 #209 №424274 
>>424273
Получается, это будет стационарная орбита Луны,но не в сфере её тяготения ? Забавная штука.
Аноним 18/09/18 Втр 01:23:00 #210 №424276 
>>424274
Получается, стационарная орбите вокруг Луны невозможна впринципе
Аноним 18/09/18 Втр 09:55:33 #211 №424407 
>>424268
>Znacheniye znayesh?
зе мля
Аноним 18/09/18 Втр 10:01:49 #212 №424409 
image.png
>>424269
>селена стационарная
Тут опять же будет ЗНАЧЕНИЕ ЗНАЕШЬ
Селеностационарная. Термин не употребим из-за отсутствия таковой орбиты, таки да.
На еще терминов: ареостационарная орбита, афродиоцентрическая ака ситериоцентрическая, жовицентрическая, где Юнона катается, ПОСЕЙДОНОЦЕНТРИЧЕСКАЯ, где еще никого не было. И мэни мэни моар.
Аноним 18/09/18 Втр 10:33:10 #213 №424411 
1426743927516432824.jpg
Недавно Интерстеллар пересмотрел и так не понял этого. Что вокруг чёрной дыры, блять? Горизонт событий? И почему он светиться? Это частицы света или что?
Аноним 18/09/18 Втр 10:35:12 #214 №424412 
>>424411
>светиться
Да ты же даже русского языка не знаешь, лул.
Он светится
https://ru.wikipedia.org/wiki/Аккреционный_диск
И вокруг ЧД светится за счет искажения гравитацией.
https://www.youtube.com/watch?v=S6qw5_YA8iE
Аноним 18/09/18 Втр 10:35:20 #215 №424413 
>>424411
Аккреционный диск.
Аноним 18/09/18 Втр 13:26:04 #216 №424438 
>>424411
Вокруг дыры аккреционный диск. Выглядит он так (как "ореол"), потому, что та его часть, которая за ЧД, для наблюдателя "линзируется" гравитацией ЧД. Физически же (по расположению вокруг ЧД) он как кольцо у Сатурна. Светится он, потому что состоит из раскаленного газа и прочих говен. Раскаляет его гравитация ЧД, точнее, её градиент у горизонта событий.
Аноним 18/09/18 Втр 13:31:29 #217 №424439 
>>424438
> Раскаляет его гравитация ЧД, точнее, её градиент у горизонта событий.
Гравитация ничего не раскаляет, раскаляет его трение в газе.
Аноним 18/09/18 Втр 13:32:29 #218 №424440 
>>424412
>>424413
>>424438
Ясно теперь, спасибо.
Аноним 18/09/18 Втр 13:45:49 #219 №424443 
ПОясните, я читал что есть водородные двигатели, которые во всем лучше ДВС, но они якобы могут взорваться и использорваться террористами - поэтому ихз не юзают в авто. Но почему их не сделают в самолёты, поезда и общественный трансопртН?
Аноним 18/09/18 Втр 13:48:17 #220 №424444 
Screenshot20180918-164734.png
>>424443
> поезда
Вчера сделали.
Аноним 18/09/18 Втр 13:51:17 #221 №424445 
>>424444
А в чем вообще плюсы и минусы этих двигателей и чсем они отличаются от классических двс?
Аноним 18/09/18 Втр 14:03:51 #222 №424450 
>>424439
Да, спасибо анон, немного ебано выразился.
Аноним 18/09/18 Втр 14:20:00 #223 №424454 
>>424440
Спасибо, что понял. Задавай ещё вопросы.
Аноним 18/09/18 Втр 14:37:01 #224 №424460 
>>424454
Можно, да? Ладно. Мне ещё не совсем понятно, что такое время и как оно может искривляться/замедляться.
1. Насчёт замедления. Взять например гравитацию, как она может влиять на время? Ведь чем сильнее гравитация у объекта какого нибудь, время возле него замедлится же. Я могу понять как гравитация может пространство искривить, но вот время...
Да и вообще, что касается тех вещей которые нельзя увидеть, пощупать и т.д, меня ставит это в тупик.
2. И как вообще время появилось? До большого взрыва не было времени я так понимаю, как и пространства. Где оно было? Я так понимаю, что время и пространство очень тесно связаны и пространство не может без времени существовать?
Аноним 18/09/18 Втр 14:55:46 #225 №424466 
15372559903860.png
>>424411
>светиться
Аноним 18/09/18 Втр 17:28:45 #226 №424483 
>>424466
Турбопроигрыш по спейсански
Аноним 18/09/18 Втр 17:30:26 #227 №424485 
>>424466
>>424483
Ну тупые.
Проверочное слово - КРЕАЦИЯ
АКРЕАЦИОННЫЙ
Аноним 18/09/18 Втр 18:03:57 #228 №424503 
>>424485
Ебать дебил. Проверочное слово крестец, потому что окрестцовый диск находится совсем рядом с полной жопой.
Аноним 18/09/18 Втр 18:04:51 #229 №424506 
xlosbc6ge5p01.png
>>424503
Почему-то неиллюзорно приграл. Даже не просто выдохнул через нос больше обычного, а посмеялся.
Аноним 18/09/18 Втр 18:09:46 #230 №424509 
Чому у Союза только один купол, а не три как у Ололона?
Аноним 18/09/18 Втр 18:18:12 #231 №424513 
Если Земля будет крутиться вокруг своей оси в 2 раза быстрее, изменит ли это жизнь на этой говнопланете?
Аноним 18/09/18 Втр 18:25:28 #232 №424516 
>>424513
Изменит.
Незначительно.
Аноним 18/09/18 Втр 18:50:49 #233 №424523 
>>424460
2. Времени нет. Точнее оно есть только с точки зрения твоего восприятия. Это 4-е измерение - как длина и ширина, так что вопрос о времени ДО бб не имеет никакого смысла.
Аноним 18/09/18 Втр 21:05:36 #234 №424542 
>>424264
>геостационарной
>гео
Значение знаешь? А если подсчитать, то селеностационарная орбита была бы вне сферы лунной сферы Хилла, так что нет.
Аноним 18/09/18 Втр 21:08:01 #235 №424543 
>>424443
Водород говно, плотность в газообразном виде OCHE маленькая.
Аноним 18/09/18 Втр 21:12:18 #236 №424545 
>>424542
Пацаны, я слоупока поймал! Что с ним делать-то?
>>424268
>>424267
>>424273
Аноним 19/09/18 Срд 01:51:47 #237 №424600 
>>421803 (OP)
Имелся ли шанс на удачный пилотируемый облёт Венеры Аполлоном в 70-х годах ?
Аноним 19/09/18 Срд 09:45:53 #238 №424633 
>>424600
Да, но не в трехместном варианте.
Технически можно одного огурца было послать на облет. Он бы охуел, и потому так не стали делать, это только недавно мы начали увеличивать время в невесомости, и только щас люди по полгода в невесомости проводят, тот бы мог оказаться неготовым к такому дерьму, особенно учитывая что не знали про как это переносить.
Аноним 19/09/18 Срд 18:06:21 #239 №424787 
>>424600
зойчем только непонятно
Аноним 19/09/18 Срд 19:06:27 #240 №424823 
А бывают узкоглазые кошки?
Аноним 19/09/18 Срд 21:01:34 #241 №424904 
Если по информации в википедии говорится что количество галактик во вселеной около 2х триллионов, можно ли сделать теоретическое предположение того что в каждой галактике есть или было или будет планета в которой есть жизнь ? необязательно в том представление жизнь похожая на нашу, возможно что и в других формах и касках что мы не можем себе представить? когда мы встретим уже наконец настоящих инопланетян ? они есть? обнаружения другой жизни вопрос времени?
Аноним 19/09/18 Срд 23:43:57 #242 №424944 
>>424904
>можно ли сделать теоретическое предположение
Сделал
Аноним 20/09/18 Чтв 03:29:47 #243 №425005 
1472298509120321612.jpg
1) пикабуГугл говорит, что вот эта хуйня на пике - искаженное отображение Земли и вообще бред чьего-то воспаленного мозга. Что реальная поверхность Земли - таки не страшный булыган, а почти идеальная сфера и перепады по высоте составляют всего +-100 метров. Эллипсоид Земли (сам по себе настолько близкий к сфере, что это невозможно заметить невооруженным взглядом при любом масштабе) с этими перепадами и называется геоидом. Геоид - это поверхность Земли на уровне мирового океана. Это так?

2) Как называется плоскость фигуры (и есть ли такие модели) в виде Земли, но без воды? Т.е. только поверхность суши?

3) Почему сфера Хилла называется сферой? Разве астрономические объекты имеют идеально равномерное гравитационное поле? Почему стоит только ляпнуть, что Земля имеет форму шара, как тут же вырисовываются куча умников и начинают с огнем и вилами в глаза рассказывать, что это не так, но про сферу Хилла никто не заикается.

4) Можно ли утверждать, что в общем случае чем массивнее астрономический объект, тем его фора ближе к шару? Или в первую очередь играет роль его плотность или структура или еще чего?

5) Как называется плоскость вращения одних объектов относительно других? Планет вокруг Солнца? Ну или аналог эклиптики по отношению к другим объектам.

5) Астрономические объекты шире по плоскости вращения чем по собственной оси из-за действия центробежной силы? Не помню какая, но одна (или несколько?) планет СС имеют заваленную почти до плоскости вращения вокруг Солнца собственную ось вращения, эта планета так же сплюснута по своим заваленным полюсам? Эта ось вращения перпендикулярна траектории движения планеты вокруг Солнца? А могут быть такие естественные астрономические объекты, у которых ось вращения лежит параллельно и они (объекты) находятся в относительно стабильном положении?


6)
Аноним 20/09/18 Чтв 03:32:51 #244 №425006 
>>424904
>когда мы встретим уже наконец настоящих инопланетян ?
Не стоит ожидать что они к нам прилетят. Есть такой универсальный закон сыча: "Что бы найти девушку надо выйти из дома". Ну а найдёшь ли ты их уже другой вопрос.
>обнаружения другой жизни вопрос времени?
Как уже сказал если просто сидеть на жопе то нет. Надо хотя бы долететь до другой системы, что бы найти её. Ну или в крайнем случае прослушивать всю сферу наблюдения, во всех диапазанах, 24\7, в надежде что в радиусе 50 св лет есть разумная жизнь. За неразумной придётся самому лететь. А не так как сейчас 1,5 угловых граздуса раз в пол года, по сугубо научным вопросам.
>они есть?
Технически космонавты сейчас живут вне земли.

>>424823
Всякая кошка при свете дня имеет узкие глаза.
Аноним 20/09/18 Чтв 03:41:02 #245 №425007 
>>425005
>2)
плоскость поверхности фигуры /фикс

>6)
пока сам не осилил

>>425006
Я выше спрашивал про глаза, а не зрачки. Бывают породы кошек с глазами, как у азиатов?
Аноним 20/09/18 Чтв 04:01:53 #246 №425008 
Вода.jpg
>>425005
1) +-12 километров при 6000 километровом радиусе, это достаточно мало и оптически ты не различишь гору эверест на общем фоне.
Геоид это по сути форма земли. А с водой или без воды это уже не столь важно. По сути это приплюснутая сфера. Но можно и геоидом назвать.
3) даже если это не идеальная сфера, то из теории пуанкаре гласит что твой пик отображения отображающий гравитационную дифференциацию земли тоже сфера.
4) Согласно современным представлениям объект из камня больше 600 километров и из льда больше 400, становится сферой. Да, чем больше гравитация, тем горы ниже.
5) Есть эклиптика земли, есть галактическая эклиптика солнца, есть эклиптика юпитера. О чём ты?
5) Если луна постоянно повёрнута к земле лицом это не значит, что она не вращается.
Любое тело будет вращатся. Где-то слабо, где-то сильно. Такие массивные объекты как планеты в основном вращаются быстро, отчего сплющиваются, пусть и не сильно. Вращение же по орбите никак не может увеличить какую-то приплюснутость. Но приливной захват поднял высоту видимой стороны луны по отношению к невидимой. Не более.

6) Половина вопросов о терминологии, а не о космосе.

>>425007
Ни одна кошка не имеет форму глаз человека. Кошка имеет форму глаз кошки, не голубя и не рыбы, а кошки.
Аноним 20/09/18 Чтв 04:07:16 #247 №425009 
>>425008
> гравитационную дифференциацию земли
Сильно преувеличенную гравитационную дифференциацию земли.
Очевидный фикс.
Аноним 20/09/18 Чтв 04:16:31 #248 №425011 
>>425009
Сильно преувеличенную дифференциацию гравитации земли.
Ещё один.
Аноним 20/09/18 Чтв 04:23:35 #249 №425013 
>>424138
песец. спасибо
Аноним 20/09/18 Чтв 16:25:25 #250 №425137 
>>421803 (OP)

Теория мультивселенной реальна?
Аноним 20/09/18 Чтв 16:50:16 #251 №425142 
Когда астероид на подковообразной орбите кувыркается в точках лагранжа земли, он получается занимает и возвращает энергию орбите Земли? Она не пропадает?
Аноним 20/09/18 Чтв 16:53:07 #252 №425143 
1527526677098.png
1. Какой мощности должен был быть удар по Урану, чтобы нагнуть его так сильно? Он же на боку катается.

2. Мог ли этот удар сорвать с него кусок поверхности и отправить его в космос?

3. Можно ли сейчас найти алмазный астероид, который когда-то был частью коры Урана?

4. Такой же вопрос, но про астероиды с кусками Венеры.
Аноним 20/09/18 Чтв 18:33:45 #253 №425154 
Прошу прощения за вторжение.

1) Насколько верно следующее утверждение "До Большого Взрыва существовала гравитация и время"?
2) У гравитации есть свойства? Может ли она изменяться под воздействием исключительно времени?
Аноним 20/09/18 Чтв 18:52:44 #254 №425155 
>>425137
Ее нельзя считать таковой, пока не найдется способа ее проверить (например, увидеть следы пекафейсов в реликтовом микроволновом фоне, если теория предскажет, что другие вселенные должны проявлять себя именно так). В любом случае, математически разработанного варианта такой теории (в том смысле, как ОТО или Стандартная модель в квантовой теории поля) пока нет.
Аноним 20/09/18 Чтв 18:59:05 #255 №425156 
>>425155
>увидеть следы пекафейсов в реликтовом микроволновом фоне
Что делать, если я вижу следы пекафейсов в реликтовом фоне?
Моему диагнозу поможет, если я скажу, что йобы вообще везде вижу?
Аноним 20/09/18 Чтв 19:01:47 #256 №425157 
>>425143
1,2,3 Массой примерно с Землю. Вряд ли, скорости в этой области СС слишком малы, чтобы что-то покинуло гравитационный колодец Урана. Но столкновение могло произойти еще на этапе формирования, когда Уран был меньше и ближе к Солнцу.
4 Венерианские метеориты вроде бы известны, гугл ит.
Аноним 20/09/18 Чтв 19:08:48 #257 №425160 
>>425156
https://en.m.wikipedia.org/wiki/Pareidolia
Аноним 20/09/18 Чтв 19:16:32 #258 №425162 
nd7EPK2y.png
>>425160
>Парейдолии часто возникают в инициальных стадиях острых психозов
Бля...
Аноним 20/09/18 Чтв 20:19:52 #259 №425170 
P0349-350-351.jpg
Как так вышло, что после войны Британия активно занималась космонавтикой, а сегодня их место заняли лягушатники, которым победу в ВМВ "подарили"?
Аноним 20/09/18 Чтв 20:46:28 #260 №425179 
>>425170
Боюсь такое говорить вслух, но видимо победа или проигрыш в ВМВ не влияет напрямую на развитие космонавтики - по крайней мере если ты не тот, кто спиздил фон Брауна.
Аноним 20/09/18 Чтв 20:57:06 #261 №425186 
>>425155

Короче представить это нереально
Аноним 21/09/18 Птн 01:54:24 #262 №425247 
>>425170
Что на пике?
Аноним 21/09/18 Птн 06:07:16 #263 №425261 
>>424445
Теоретически водород бесконечный вообще, нужна только энергия чтобы его добыть любым из десятков, а то и сотен способов.
Аноним 21/09/18 Птн 10:38:45 #264 №425292 
15366024619620.png
Поясните, я видел всякие видео, где в камере из куска металла вытачивают всякие йобы.
Почему так не делают из монолитного куска металла двигатели для ракет? Ведь в этом случае они были бы убердешевыми и можно их было бы штамповать сотнями.
Аноним 21/09/18 Птн 10:46:11 #265 №425293 
15374700847590.jpg
И еще вопрос, почему при запуске ракет не делают вспомогательные опоры для запуска? Т.е. чтобы сэкономить на начальном ускорении ракету можно было бы подбрасывать.
Аноним 21/09/18 Птн 10:49:13 #266 №425295 
>>425292
Много мелких деталей.
>>425293
Нихуя это не даст.
Аноним 21/09/18 Птн 11:04:10 #267 №425298 
>>425295
Так пусть делает и мелкие детали, а роботы всё собирают на конвейере. Вот тебе и десяток ракет в день.
>>425295
>Нихуя это не даст.
Чову?
Аноним 21/09/18 Птн 11:07:40 #268 №425300 
>>425298
>делает и мелкие детали, а роботы всё собирают на конвейере
А как по твоему их делают?
>Чову?
Потому что 5 метров вверх нихуя не даст когда нужно взлететь на пару сотен км и набрать огромную скорость вбок.
Аноним 21/09/18 Птн 11:12:26 #269 №425301 
>>425300
>А как по твоему их делают?
Ну ракеты я думал штучный экземплятор, и роботы только основы делают какие-нитбудь.
Аноним 21/09/18 Птн 11:30:19 #270 №425304 
>>425247
Краткая часть плана развития московского метрополитена на 2040 год.
Аноним 21/09/18 Птн 11:33:26 #271 №425305 
Сап, анон. Какая последняя актуальная инфа по минимальному размеру вселенной?
Аноним 21/09/18 Птн 14:15:40 #272 №425378 
>>425305
Ну километров 30 будет как минимум.
Аноним 21/09/18 Птн 14:16:36 #273 №425379 
Так что там с атмосферой сатурна? Кассини упал, а где телеметрия?
Аноним 21/09/18 Птн 14:20:53 #274 №425382 
>>425379
Че успел - передал, но он в верхних слоях сгорел. Туда надо или пробку с долгим падением на парашюте и йоба-теплощитом чтобы при входе еще не сгорел, либо зондировать приборами не залетая в нее, чем, собсно, Касиня и занимался. По сравнению с тем, что он ловил на отламывающиеся антенны и сгорающие приборы он об атмосфере больше узнал этими приборами на расстоянии от Сатпппрна.
Аноним 21/09/18 Птн 19:16:55 #275 №425553 
а зачем надо объединять все четыре фундаментальные силы в одну, связывать их, чтобы один закон был или чо там ученые пытаются много лет сделать? почему не развивать науку отталкиваясь от того, что законы разные, и они вот такие, и работают отдельно, и типа так и должно быть? какие профиты от того что будет там единая теория всего?
Аноним 21/09/18 Птн 19:50:28 #276 №425564 
>>425553
Есть вася он покупает хуи. Есть лена и она покупает пёзды. Можно конечно рассматривать отдельно то что вася покупает хуи, а лена пёзды, и нихуя не понять зачем вася покупает хуи, но когда выяснится, что и лена и вася это менеджеры в ЕБЛЯ-инкорпарейтед, становится ясно что это не просто покупатель вася и покупатель лена, а покупатель ЕБЛЯ инк.
По сути ты можешь просто оперировать силами трения, силами упругости, и тебе не важно знать что за взаимодействия такие, и современная теоретическая физика в отличии от практический, не отвечает на вопрос "как", но отвечает на вопрос "почему". Разбирают все перепетии всяких глюонов, хуитонов и прочих, как, что и с чем. Когда выяснят все перепетии это и будет готовая теория всего. У нас есть худо-бедная часть описывающие взаимодействие 3х сил, и пока искали связь с гравитацией, открыли ещё и бозоны с 5 взаимодействием.
Аноним 21/09/18 Птн 19:53:13 #277 №425567 
4545.png
и еще вопрос ядро атома состоит из протонов, нейтронов, то есть образно на бумаге это выглядит так: атом шарик - ядро шарик - шарики протоны, нейтроны, или атом шарик - ядро и есть шарики протонов и нейтронов? как бы вот у нас атом на бумаге это круг да? внутри этого круга будет еще круг в котором уже будут протоны нейтроны или же нет никакого круга, а ядро атома это и есть совокупность протонов нейтронов? короче пикрил первый вариант или второй?
Аноним 21/09/18 Птн 19:58:43 #278 №425572 
>>425564
а бля я и забыл что всякие упругости и трения существуют) они кстати куда относятся? гравитация?
Аноним 21/09/18 Птн 20:07:20 #279 №425573 
>>425572
Практически все силы, которые в обычной жизни действуют, относятся к электромагнитному взаимодействию, в том числе и трения, и упругости, и хуюгости, все взаимодействия между атомами — электромагнитные.
Аноним 21/09/18 Птн 20:20:09 #280 №425580 
14883062505100.jpg
>>425567
>атом
Всё намного сложнее, чем какие-то шарики. Для описания структуры атома, грубо в виде списка, можно перечислить электрон, нейтрон, протон. Для химии этого достаточно. И в повседневной жизни ты вряд ли столкнёшься с нейтрино(лол). Но для понимания сути происходящего надо думать и вникать в описание этой вашей стандартной модели. И там нет никаких кружков, шариков, и в большинстве своём логики макромира.
Аноним 21/09/18 Птн 22:12:32 #281 №425617 
15371713635712.jpg
зачем в двигателях жидкотопливных ракет топливные насосы? Чому бы не использовать самоток топлива плагодаря сначала силы притяжения земли. а в космосе не допускать замедления, чтоб топливо не отступило, тем же горячим разделением ступеней
Аноним 21/09/18 Птн 22:14:17 #282 №425618 
>>425617
Потому, что самотоком ты не сделаешь сотню атмосфер давления, ты че.
Аноним 21/09/18 Птн 22:16:01 #283 №425619 
>>425618
а зачем так много сотен атмосфер на подачу топлива?
Аноним 21/09/18 Птн 22:17:04 #284 №425621 
>>425619
Потому, что ты хочешь чтобы у тебя была реактивная ракета выбрасывающая сотни тонн за минуту, а не газовая гарелка как у анона-омича из соседнего треда, которая дает тягу чуть больше, чем давление в баллонах.
Аноним 21/09/18 Птн 22:21:13 #285 №425622 
>>425621
ну чисто теоритически давления столба жидкости чо представляет из себя ракета должно хватить чтоб топливо через лейку по принципу дачного душа поступало в камеру сгорания а дальше та тяга что развилась будет толкать ракету, ускорять, G растут топливо больше идет
Аноним 21/09/18 Птн 22:24:43 #286 №425623 
>>425622
Ты там подожжешь в камере сгорания, оно и будет ГОРЕТЬ.
И будет так ракета стоять минуты и снизу из сопел неспешно факел как на нефтяной платформе ебашить, пока не прогорит какой-нибудь провод или бак и не будет спектакулярный выброс энергии/незапланированная быстрая разборка.

>ну чисто теоритически давления столба жидкости чо представляет из себя ракета должно хватить
Вот какой теорией ты руководствуешься, чтобы такое говорить? Попробуй честно посчитать, что ты пытаешься себе за правду внушить.
Все цифры есть, формулы подскажем.
Аноним 21/09/18 Птн 22:28:17 #287 №425624 
>>425623
ну хуй с ним подавать на старте в камеру сгорания топливо из отдельных баков мелких но наддутые для старта с зеемли до сотни атмосфер, так на этапе старта будет секунд 10 йоба тяги а дальше уже G помогут
Аноним 21/09/18 Птн 22:29:34 #288 №425625 
>>425624
Топ лел.
Ты только что вытеснительную подачу.
В некоторых реализациях она и не требует ТНА.
Аноним 21/09/18 Птн 22:33:12 #289 №425626 
>>425625
ну йопту чому этого нет в ракетах носителях и балистических? деньги не все еще попилили? Удешивлять ракеты пролетареату кто будет?
Аноним 21/09/18 Птн 22:34:38 #290 №425628 
>>425626
Вытеснительная подача есть в ракетах.
Только она ХУЖЕ турбонасосных агрегатов, поэтому в основном используют их.
Даже ньюфаги вроде Электрона, где вместо сжигания в камере ТНА крутят его электромотором.
Аноним 21/09/18 Птн 22:37:27 #291 №425630 
>>425628
чем хуже?
Аноним 21/09/18 Птн 22:37:57 #292 №425632 
>>425630
Чем турбонасосная.
Аноним 21/09/18 Птн 22:55:17 #293 №425643 
>>425624
Баки получаются тяжелыми. Компоненты топлива могут менять свои физические/химически свойства при большом давлении. И нужно не просто высокое давление в баках, а близкое к равномерному на всем участке полета. Иначе у тебя здорово пыхнет на старте и по мере опорожнения баков давление начнет катастрофически падать, начнет падать тяга.
Аноним 22/09/18 Суб 00:06:22 #294 №425675 
У шаттла >>425672 крылья пустотелые были? Там нету ничего кроме стойки шасси и элементов управления рулями а еще тысячи датчиков и прочей ерунды? А у бурана?
Аноним 22/09/18 Суб 00:09:05 #295 №425677 
BlueprintSpaceShuttle Wing.png
BlueprintSpaceShuttleOrbiter-Illustration.jpg
BlueprintShuttleFusStructISO2000.gif
>>425675
Там каркас внтуре, ёба. А так да, примерно как ты сказал. У Бурана то же самое.
Аноним 22/09/18 Суб 00:12:20 #296 №425679 
tmz-0.jpg
kks.jpg
wings.jpg
>>425677
Вот Бурановские крылышки KFC кстати.
Аноним 22/09/18 Суб 00:12:44 #297 №425680 
image.png
image.png
>>425675
Это ж не самолёт, там баки любой формы не напихаешь, они должны лютое давление выдерживать, плюс перегрузоньки туда-сюда.
Вот будешь такой входить в атмосферу пару раз покувыкравшись с МКС, а у тебя солярка в левое крыло залилася, и ты теперь повторяешь подвиг Колумбии.
Крылья и так большой вес, туда мелкобаки только мешать станут, профита от них по сравнению с цилиндрическими и сферическими нема.
У других космопланов такая как раз планировочка, что солярка в центроплане см Скайлон, например. Потому Венчурстар хотели по схеме несущего фюзеляжа (или как там оно зовется, забыл уже) делать, чтобы подъемную силу таки жырными (и легкими) баками создавать
Аноним 22/09/18 Суб 00:13:18 #298 №425681 
horribly memed.jpg
>>425679
>Бурановские крылышки KFC
Щас опять обидно было.
Аноним 22/09/18 Суб 00:18:15 #299 №425684 
tmz-7.jpg
>>425681
Ой да ладно тебе, если бы я про Шаттл так пошутил, было бы куда жёстче.
экипаж STS-107.жпг
>опять
Абажжи, а до этого когда было? inb4 когда МИК обрушился
Аноним 22/09/18 Суб 00:20:53 #300 №425685 
>>425684
>Ой да ладно тебе, если бы я про Шаттл так пошутил, было бы куда жёстче
Точняк. Чето я про загубленную потерянную программу челноков запереживал забыв, что у работавшей 14 огурцов подружилось с Кракеном.
В любом случае, триггернулся на жаренные крылышки космоплана, прям по живому.

>Абажжи, а до этого когда было?
Да то ж постоянно уопминают Буран на роскосмотреде, то и дело гореть начинаю как экипаж Колумбии Будь я проклят.
Аноним 22/09/18 Суб 00:54:28 #301 №425711 
14384355784360.jpg
Если покинуть сферу притяжения Солнца, то мы попадем на гигантскую орбиту вокруг центра галактики? Это вообще может считаться орбитой со свойственными ей апоцентром и перицентром?

А если выйти из сферы влияния гравитации галактики, то что тогда? Мы окажемся на какой-то орбите вокруг великого аттрактора, или же мы просто будем падать куда-то на какой-то гравитационный центр? Или же мы окажемся на вытянутой кометной орбите вокруг какого-то далекого гравитационного центра и даже упасть в него будет невозможно потому что он будет еще куда-то двигаться и удаляться от нас, в то время как мы все равно будем оставаться его гравитационной сучкой? Можно ли погасить все скорости относительные, чтобы не быть ничьей гравитационной сучкой и чтобы каждая точка вселенной удалялась конкретно от нас равномерно во все стороны?
Аноним 22/09/18 Суб 00:59:34 #302 №425716 
>>425711
>Если покинуть сферу притяжения Солнца, то мы попадем на гигантскую орбиту вокруг центра галактики? Это вообще может считаться орбитой со свойственными ей апоцентром и перицентром?
Да.
>А если выйти из сферы влияния гравитации галактики, то что тогда?
А куда-то в ебеня полетишь.
>Мы окажемся на какой-то орбите вокруг великого аттрактора, или же мы просто будем падать куда-то на какой-то гравитационный центр?
Нет. Доминирующее притяжение за галактикой - локальная группа.
Аттрактор просто показывает движение групп. На деле расширяется вселенная быстрее, чем падаем в аттрактор, так что он для показа макроскопической структуры, а не "куда все падает".
>Можно ли погасить все скорости относительные, чтобы не быть ничьей гравитационной сучкой и чтобы каждая точка вселенной удалялась конкретно от нас равномерно во все стороны?
Тебе даже особо летать не надо. Погодь триллиончики лет и если расширение будет продолжаться, то вскоре окажется, что ничего никуда милкдромеду не притягивает, других галактик не видать и их влияния тоже.
Потом еще немного - и других звезд не будет, только наша СС во всей вселенной.
А потом глядишь - и солнца нет, сычуешь на камне (ну той звезды что ты солнцем тогда выберешь).
А потом и глазом моргнуть не успеешь - все частицы из всего что состоишь перестанут быть связаны.
Аноним 22/09/18 Суб 01:07:54 #303 №425723 
>>425716
>Доминирующее притяжение за галактикой - локальная группа.
Вот я и не могу представить себе орбиту в голове, которая вокруг этого крутится, потому что в этой локальной группе нет четкого центра, он же как-то смещается периодически наверное, как орбиту нарисовать в голове вокруг этой аморфной кучи? Как орбиты прорисовывается на мировом серваке под названием Вселенная, если гравитация до нас доходит со скоростью света и когда мы на орбите совершаем какое-то трек, а центр гравитации уже сместился, там барицентр подскочил на 10 парсек выше блядь, или там 2 галактики слились и всё поменялось, что с моей орбитой вокруг этого локального центра происходит?
Аноним 22/09/18 Суб 01:21:28 #304 №425724 
В ходе проверки внешней обшивки МКС, на соскобах с поверхности корпуса и иллюминаторов были обнаружены следы жизнедеятельности морского планктона. Из википедии.

Вопрос: откуда в космосе планктон?? Или я совсем тупой..
Аноним 22/09/18 Суб 01:23:20 #305 №425726 
>>425724
Дальше читать не пробовал?
Микроорганизмы дохуя мелкие, восходящими потоками и прочим поносом возгоняются до стратосферы и иногда как и молекулы воздуха гоняют на суборбиту, с которой, обычно и возвращаются, но вот эти вот наебнулись о мимопролетавшую станцию.
Аноним 22/09/18 Суб 02:22:52 #306 №425745 
Что такое астероид и комета?
В википедии ничего нету, удивительно просто.

Астероид - каменистое тело, комета - ледяное? Пресловутый хвост кометы, про который в школе рассказывали, не обязателен? Комета же может быть не только вблизи Солнца?
Аноним 22/09/18 Суб 03:09:08 #307 №425747 
15362250747880.jpg
>>425745
>В википедии ничего нету, удивительно просто.
Аноним 22/09/18 Суб 05:17:56 #308 №425752 
там какой то ученый гипотезу римана доказал в понедельник пойдет рассказывать

это круто? какое практическое применение?

а еще вопрос где то слышал что самая минимальная частица пространства или чот подобное это планковская длина типа меньше быть не может но что нельзя просто ее разделить и посмореть что внутри нее? эта планковская длина тоже из чего то должная состоять а та штука из чего то еще и так до бесконечности я просто не понимаю как может быть что-то элементарное неделимое состоящее из ничего
Аноним 22/09/18 Суб 07:15:20 #309 №425755 
unnamed.jpg
>>425382
>ловил на отламывающиеся антенны и сгорающие приборы
Аноним 22/09/18 Суб 09:54:40 #310 №425774 
>>425752
Нет, это невинная шутка математика. Он же глубокий старичок.
Аноним 22/09/18 Суб 12:54:28 #311 №425800 
>>425752
>где то слышал
Хуёво слышал.
Это не минимальная длинна, а минимальная длинна имеющая физический смысл. Есть например выводы, о том что зернистость мира, если она есть, не больше чем, на 20 с хвостом порядков меньше, чем ПД. Ничего не запрещает быть объектам меньшим, чем ПД.
Аноним 22/09/18 Суб 12:57:04 #312 №425803 
>>425800
>минимальная длинна имеющая физический смысл.
Что это значит?
>на 20 с хвостом порядков меньше, чем ПД. Ничего не запрещает быть объектам меньшим, чем ПД.
Тогда в чем смысл этих вещей?
Аноним 22/09/18 Суб 13:19:19 #313 №425810 
>>425800
>длинна

Повбывав бы.

Смысл ее в том, что при попытке измерить длину меньше планковской необходимо согласно квантовым соотношениям неопределенности использовать частицы, взаимодействующие с энергиями, достаточными для их коллапса в миниатюрную ЧД. Размер горизонта ЧД будет чуть больше планковской длины. Следовательно, вопрос о длинах меньше планковской ненаучен, т.к. такие длины ненаблюдаемы.
Аноним 22/09/18 Суб 13:22:50 #314 №425812 
image.png
>>425810
Я нихуя не понял.
Это типа если фотон обладает такой длиной волны, то в нем так много энергии, что по принципу е=мс2 и массы, и эта масса в объеме фотона схлопывается в черную дыру что ли?
Аноним 22/09/18 Суб 13:28:30 #315 №425814 
>>425812
>е=мс2
Нет такого принципа для фотона. Эквивалентность энергии массы. Есть аш*ню.
Аноним 22/09/18 Суб 18:16:48 #316 №425958 
Не могло ли так получиться, что материи и антиматерии все-таки поровну и просто в момент когда вселенная стала менее плотной, материя и антиматерия сгруппировались в отдельные галактики которые друг с другом не обмениваются веществом от чего нет наблюдаемой аннигиляции и вся аннигиляция давно прошла?
Какие есть пруфы того, что это не так?
Аноним 22/09/18 Суб 19:46:27 #317 №425979 
NGC6052[1].jpg
Whirlpool(M51)[1].jpg
Thelastwaltz[1].jpg
>>425958
Например, то, что галактики не статично в небе подвешены, а друг с другом сталкиваются, пиздят друг у друга газ и обмениваются веществом еще как, но даже полшишечки аннигиляционного излучения ни разу не было зафиксировано.
Аноним 22/09/18 Суб 23:47:40 #318 №426053 
Правда, что в 90-ые или 00-ые уебали какую-то АМС поставит вверх ногами акселерометр размером с рублевую монетку?
Аноним 22/09/18 Суб 23:51:23 #319 №426054 
image.png
image.png
image.png
image.png
>>426053
С Протоном не путаешь?
https://www.youtube.com/watch?v=EJ5__1PPgNQ
Аноним 23/09/18 Вск 00:08:31 #320 №426057 
>>426054
нет, амс была, то ли японская, то ли американская
Аноним 23/09/18 Вск 00:16:15 #321 №426059 
>>426057
https://en.m.wikipedia.org/wiki/Genesis_(spacecraft)
Аноним 23/09/18 Вск 00:17:35 #322 №426061 
>>426059
Нахуя ты мобильную ссылку постишь, пидрила?
Исправил:
https://en.wikipedia.org/wiki/Genesis_(spacecraft)
Аноним 23/09/18 Вск 00:56:32 #323 №426064 
Вот представим у возращаемого АМС отказал парашют, насколько эффективно тормозить оставшимся гидразином дабы уебатся менее жёстко?
Аноним 23/09/18 Вск 01:02:35 #324 №426066 
>>426064
Нинасколько, тяговооружённости не хватит.
Я даже не уточняю про что речь и про какой полёт, такое применимо к любым случаям.
Аноним 23/09/18 Вск 01:08:23 #325 №426068 
>>426066
>не уточняю про что речь и про какой полёт, такое применимо к любым случаям.
нууу, такое, вот недавно был разговор про уебанство Венеры-9(Космос-482) об Землю и многие анону говорили, что части ценные выживут, потому что оно тормозит медленно годами и плюс приспособлено к венере.
Хотя думаю анон задавший вопрос не это имел ввиду.
Аноним 23/09/18 Вск 01:35:22 #326 №426072 
>>426068
>>426066
Имел ввиду, что вот падает аппарат с ожидаемой конечной скоростью столкновения условные 2н. Вот мы резко врубаем гидрозин на максимум вниз и сокращаем скорость падения до 1,5н. В первом случае какая-нибудь камера с образцами лопнет и разлетится, во втором случае имеет шанс что даже уцелеет. Подобно тому когда в огурцах аварийно приземляешься без парашута и крыльев.
Аноним 23/09/18 Вск 01:37:34 #327 №426074 
>>426072
>ввиду
>гидрозин
>парашута
Чето расхотелось объяснять даже.

Ты сократишь не с 2н до 1.5н, а с 2н до 1.99н, у аппаратов не ставят маршевые движки с ТВР у поверхности даже близко к единице.
Аноним 23/09/18 Вск 01:52:24 #328 №426075 
relay-card.png
>>426053
Генезис уебали при возврате в 2004.
https://www.youtube.com/watch?v=dhCrOdbOUkY
Там был старый дизайн, от которого отказались. Новую плату разработали по старой документации, в которой плата должна была лежать вверх ногами относительно нового проекта. В результате акселерометрические реле пикрилейтед оказались не той стороной и в реальном железе. Это было пропущено и разработчиками, и ревью, и приёмкой, и "красным" отделом (который занимается выявлением проблем в процессах разработки). Проблема нестыковки документации, в общем - типичная хуйня, по этой причине происходит наверно 90% проблем на АМС. Например фейл с RUSO/TUSO у Гюйгенса, или проёб Mars Climate Orbiter из-за перевода единиц измерения между метрической и американской системами.

С разъёбанного Генезиса результаты частично спасли потом, кстати. В результате долгой и кропотливой работы.

Ещё не далее как в 2016 уебали Скиапарелли об Марс, тоже в прямом эфире, всем спейсачем следили.
https://en.wikipedia.org/wiki/Schiaparelli_EDM_lander
Сценарий очень схожий, но причина иная - две высокоуровневых ошибки проектирования: 1) недостаточно точное моделирование АМС в высокотурбулентной фазе спуска, не учтены многие факторы и 2) недостаточные диапазон ДУС и санитизация входного воздействия в СУ. В результате зонд банально болтануло, сигнал ДУС вышел за предусмотренные рамки, в результате дезориентация и потеря АМС.

>>426064
Неэффективно совершенно, т.к. тяговооруженность околонулевая, под атмосферу не заточено и дельты осталось около нуля. Но главное вовсе не это, а то что задетектить произвольный отказ изнутри отказавшей системы - задача сродни вытаскиванию самому себя за волосы из болота. В Генезисе, например, не просто "парашют отказал" (это лишь следствие), а АМС вообще не распознала вход в атмосферу. Никакое дублирование бы тут не помогло, т.к. причина отказа не в исполнительной части, а выше уровнем.

Есть неплохой научпоп по человеческому фактору в теории надёжности, советую почитать что-нибудь по этой теме, например The Field Guide to Understanding Human Error Сидни Деккера. Или по причинному анализу, тоже можно насоветовать.
Аноним 23/09/18 Вск 01:55:26 #329 №426076 
>>426072
>Подобно тому когда в огурцах аварийно приземляешься без парашута и крыльев.

блэт, реально растет поколение людей, которые думают, что если у них чот в агурцах получается, то и в жизни получится. ксп хоть и полезен, но по грубому. Я помню космическую станцию из 4 больших модулей приземлил в море на 1 парашюте и она там плавала на поверхности, полностью невредимая и вернул её со 100% деталей, но мне бы в жизни не пришло в голову, что в реальности также можно на парашюте Мир или несколько модулей МКС спасти.
Аноним 23/09/18 Вск 02:02:18 #330 №426077 
w74ZTqEs8F4.jpg
>>426076
>людей, которые думают, что если у них чот в агурцах получается, то и в жизни получится.
Если я упаду с 10-километровой высоты и приземлюсь на шлем, я смогу спокойно встать и пойти по своим делам? В огурцах прокатывало.
Аноним 23/09/18 Вск 02:46:51 #331 №426086 
Существует международной эталон температуры? Как яйцеголовые могут знать, что их градусник показывает 700 градусов по Цельсию, а не 702? Предположим, известна или принята таковой температура какого-то процесса с точностью N знаков после запятой. Как отмерить от нее ровно 1 градус?
Аноним 23/09/18 Вск 03:03:01 #332 №426087 
>>426086
https://ru.wikipedia.org/wiki/Основные_единицы_СИ
Аноним 23/09/18 Вск 03:03:27 #333 №426088 
>>426086
Абсолютный ноль — 0К ровно, -273,15°C ровно, тройная точка воды — 273,16К ровно, 0.01 °C ровно. Никаких десятков знаков после запятой, шкала температуры четко зафиксирована на этих значениях и один градус отсчитывается как 1/273,16 от расстояния между ними.
Аноним 23/09/18 Вск 03:05:41 #334 №426089 
>>426077
Ну, если ты искренне так полагаешь, то остается только пожелать удачи.
Но ты иронизируешь.
Аноним 23/09/18 Вск 13:36:28 #335 №426143 
>>426087
>пук

>>426088
>тройная точка воды
>0.01 °C ровно.
а при 0.02 что, не будет такого эффекта?

>один градус отсчитывается как 1/273,16 от расстояния между ними.
расстоянии на чем? У меня есть эталон метра, я могу приложить к нему эталонную линейку и отмерить какую-то долю. Как отмерить 1/273,16 расстояния от ТЕМПЕРАТУРЫ?
Аноним 23/09/18 Вск 13:38:54 #336 №426144 
>>426087
Ну ладно, не пук, там тоже самое, что анон выше написал. Но там нету ответа
>Как отмерить 1/273,16 расстояния от ТЕМПЕРАТУРЫ?
Аноним 23/09/18 Вск 13:44:29 #337 №426145 
А тут тупые вопросы только про космос или можно всякое спросить? А то вот где-то слышал, что когда война, то природа делает так, чтобы рождалось больше мальчиков. Это чо за пиздец? Как она это понимает? Как она осознает, что война? Типа деревья такие смотрят и такие ууу природа ебать слушай там война гитлер напал крч надо больше мальчиков рожать. Так что ли? Хз насколько это правда. Тогда почему природа не поймет, что мне на хуй не нужны волосы в жопе в 21 веке, и на хуе, и в подмышках, да вообще, на теле бля. Зато на голове мне волосы нужны именно наверху, по бокам можно и убрать, пусть андэкат будет, так сука хуй там, в очке растут активно, а вот на голове естественно выпадают причем именно блядь на верху, а не по бокам, как хотелось бы. Ебанутая тупорылая природа блядь. Хули она мне геморрой не уберет? Я в 21 веке живу блядь мне не надо много ходить все, все люди щас мало двигаются, измени мне блядь кишки, чтобы можно было не двигаться и геморроем не болеть, так хуй там. Геморрой же вроде болезнь 21 века так сказать, да еще и вроде только человек ей болеет. Но когда война рожаем мальчиков. Пизедц.
Аноним 23/09/18 Вск 14:09:39 #338 №426149 
>>426145
>А тут тупые вопросы только про космос или можно всякое спросить?
Здесь бывало спрашивали почему пельмени при варке всплывают. Но вообще-то да, про космос.

>вот где-то слышал, что когда война, то природа делает так, чтобы рождалось больше мальчиков
Вот где слышал, там и спрашивай. в /б/, видимо А после того почитай что такое систематическая ошибка в статистике, и насколько дохуя их разных бывает.
Аноним 23/09/18 Вск 14:14:19 #339 №426150 
>>426077
>>426089
И всё же были случаи когда при падении с 10 км выживали.Не на шлем конечно, но всё же. А АМС вообще почти целые падают.
Аноним 23/09/18 Вск 14:18:49 #340 №426151 
>>426150
>Не на шлем конечно, но всё же.
Но хотя бы спокойно вставали и шли по своим делам?
Аноним 23/09/18 Вск 14:31:43 #341 №426152 
>>426150
Разве только с 5км.
https://ru.wikipedia.org/wiki/Столкновение_над_Завитинском

>>426151
Ну можно сказать и так, плюс-минус сапог.
https://ru.wikipedia.org/wiki/Савицкая,_Лариса_Владимировна
>Несколько часов Савицкая была без сознания. Она получила ряд серьёзных травм, но могла самостоятельно передвигаться.
Аноним 23/09/18 Вск 14:46:58 #342 №426154 
>>426152
>Разве только с 5км
А эта?
https://ru.wikipedia.org/wiki/Вулович,_Весна
Впрочем какая там нахуй разница. Что 5км, что 20 - вопрос удачного приземления. Ну чем ниже, тем больше шансов не двинуть кони от кислородного голодания.
Аноним 23/09/18 Вск 14:53:50 #343 №426155 
>>426154
Ну вон Сиболд в 2014 при катастрофе SpaceShipTwo выжил на 17км каким-то макаром, без высотки и дыхательного аппарата, на 1.2 маха. (правда с креслом и парашютом) Был ещё тот пилот SR-71, который выжил после попытки выкинуть массогабаритный макет из бомболюка и распидораса на 3.2 махах, но там все серьезно было
Аноним 23/09/18 Вск 15:09:46 #344 №426157 
Что за сети-антисети драма?
Аноним 23/09/18 Вск 15:11:41 #345 №426158 
>>426157
Есть SETI, на которую всем похуй, а антисетипетух, которому нечего делать в лечебнице, поэтому он воюет с SETI.
Аноним 23/09/18 Вск 15:29:11 #346 №426161 
>>426152
>>426154
То были в креслах, Вулович вообще планировала на хвостовом оперении (утрирую, конеш, но оно сильно удар смягчило).
ТЕРМИНАЛ
ОЧКА
смертельна.
Если какой-то приспособой терминалочку порезать и падать медленней, то шансы появляются.
Аноним 23/09/18 Вск 21:45:06 #347 №426242 
>>426149
> почему пельмени при варке всплывают
почему? Задал в поисковике, выдало с полдесятка разных ответов. Пузырьки воздуха, которые прилипают или забиваются в "поры" теста. Разложение белков при варке, муки в клейстер, субпродуктов в желатин, получившиеся продукты имеют меньшую плотность и заставляют пельмени всплыть. Что-то про насыщенный пар, давление и объем, но я не донца понял. И прочее.
Аноним 23/09/18 Вск 23:35:46 #348 №426282 
15366185123280.jpg
Посоны а взлетит ли с инжинерной точки зрения полностью реюзабельная ракета для доставки модулей лунной станции и на орбиту и на поверхность луны? Ну типа 3-4 ступени для вывода тонн 100 йоба сборку, каждая часть которой вернется на землю, из модуля станции и его доставшика, который должен разогнаться с нагрузкой в сторону луны, затормозить у нее, сесть на ее поверхность опционально, сгрузить модуль, взлететь в сторону земли и сесть на землю с последующем реюзом всего
Аноним 23/09/18 Вск 23:38:12 #349 №426285 
>>426282
Да, наркоман. С инженерной точки зрения взлетит.
С практической, а тем более с финансовой - едва ли, по крайней мере при нас, по крайней мере при текущих темпах.
Зайди в огурцач - тебе продемонстрируют, что это в теории осуществимо, циолковский не запрещает. >>424631 (OP)
Аноним 24/09/18 Пнд 09:18:03 #350 №426404 
Я тут задумался, а какие могут быть формы жизни вообще? Вот взять людишек, да и всех организмов на планете, мы ведь биологическая форма жизни, так? Так. Теоретически какие ещё могут быть? Вот например если мы возьмём Чужого, то он био-механическая форма уже, а ведь теоретически и такое может быть, чому нет. А если например возьмём планеты, может мысль глупая, но вдруг наша планета тоже живая? Ведь она живёт вроде как, без нашего ведома электромагнитные волны посылает в космос как и любая другая планета и т.д, как такую форму жизни назвать можно? Ведь для нас привычно только то, что похоже на нас, биологические организмы, а что если есть то, что выходит за рамки нашего понимания? Ведь о планете как о живом существе мало кто думает, всем похуй.
Аноним 24/09/18 Пнд 11:38:18 #351 №426434 
>>426404
>Ведь о планете как о живом существе мало кто думает, всем похуй.
Во-первых у тебя ебеная каша в терминологии и такая же в голове. Формулируй вопрос не так гуманитарно, ознакомься хотя бы с энциклопедическим базисом по вопросу. Во-вторых критерий Поппера. Придерживаясь его, размышления о всякой подобной дичи - удел фантастов. И они не терялись. Лем и его "Солярис" с организмом-океаном концептуально близок твоим омским идеям.
Аноним 24/09/18 Пнд 12:33:15 #352 №426439 
>>426434
Тут же тред тупых вопросов.
> Во-первых у тебя ебеная каша в терминологии и такая же в голове. Формулируй вопрос не так гуманитарно
Да, я гуманитариеблядь, по другому не могу.
> размышления о всякой подобной дичи - удел фантастов.
То есть, попытки взглянуть на вещи (в нашем случае формы жизни) под другим углом - это дичь по-твоему?
> И они не терялись.
А я потерялся, я не фантаст и человек далеко не умный, собственно поэтому и хочу умных анонов послушать.
> ознакомься хотя бы с энциклопедическим базисом по вопросу.
Подскажи тогда литературу/статьи какие нибудь, чтобы я ознакомился.
Аноним 24/09/18 Пнд 13:35:23 #353 №426451 
>>426439
>Гумутарий
Это не оправдание необразованности.
>Под другим углом это дичь?
Нет, под другим углом смотри сколько хочешь, проблема в том что не на что смотреть. Есть фентезийные придумки, а есть научные теории. Они никак не пересекаются.
>Я человк не умный.
Опять же это не оправдание. Любую необразованность можно исправить. Было бы желание.
>Подскажи тогда литературу
Смотря что ты хочешь, если хочешь невероятные захватывающие дух открытия, то иди в художественную литературу с меткой о научной фантастике. Заодно воображение прокачаешь и меньше будешь теряться. Если хочешь что-то из реального мира, то можешь накатить научпопа, в виде детских книжек или роликов на тытрубе. Ты конечно можешь и попытатся осилить какую-то дисциплину по харду, но по видимому тебе всё равно надо начинать с детских книжек и роликов на тытрубе, дабы хотя бы понять что это такое.
Аноним 24/09/18 Пнд 13:54:33 #354 №426455 
>>426439
>это дичь по-твоему
Не по-моему, а с точки зрения научного подхода. Можешь купаться в своих фантазиях, можешь даже оформлять их в посты в жежешечке с претензией на философию. И для кого-то они даже могут иметь ценность, ознакомят с альтернативными взглядами. Но это доска техническая, не философская. Здесь тебе ответят с постамента научной системы, и да, в таком контексте это дичь.
То есть у группы Radiohead, например, присутствует композиция под названием 2+2=5. В ней есть определенный смысл, двойное дно и т. п. Пресловутая философская ценность. Во всяких ребусах для любящих нестандартный подход предлагаются ложные доказательства тому самому "2+2=5". В данном случае эта конструкция имеет развлекательно-образовательный смысл, призвана расширять пул методик поиска истины. Имеет ценность. Но с точки зрения арифметики это именно дичь. И, скажем, преподаватель, втирающий первоклашкам, что 2+2=5 - ебаный вредитель. Смекаешь?
>Подскажи
В твоем случае достаточно бегло ознакомиться со статьей "жизнь" на википедии. Увидеть плюрализм терминов и сформировать более корректно и узко что ты хочешь знать. Сейчас у тебя уровень "хочу знать все"! Отвечаю - все знать нельзя. Приходится выбирать, что знать. Тут уж дело за тобой.
Аноним 24/09/18 Пнд 13:55:42 #355 №426456 
>>426451
> Есть фентезийные придумки, а есть научные теории
Разве нет научных теорий о существовании не только стандартной в нашем понимании жизни? Хочешь сказать, что существование какой нибудь жизни на основе кремния т.д. на какой нибудь планете это фентезийные придумки?
В общем мы друг друга вообще не поняли.
> Любую необразованность можно исправить. Было бы желание.
А зачем тогда этот тред?
Ну ты и зануда, пиздец.
Аноним 24/09/18 Пнд 13:59:27 #356 №426457 
>>426456
> Хочешь сказать, что существование какой нибудь жизни на основе кремния т.д. на какой нибудь планете это фентезийные придумки?
Это я к тому, что если есть мы, то есть и другие. Если мы биологические организмы, разве все во вселенной все организмы должны биологические быть?
И я не считаю свою фантазию чем-то плохим, и очень сомневаюсь, что если бы подобные вопросы задал действительно умному человеку, учёному, то он бы начал смеяться и язвить как ты. В общем бывай.
Аноним 24/09/18 Пнд 14:25:44 #357 №426460 
f12bd780e1ed1555a429fa78c500baf5 (1).jpg
>>426457
Большинство учёных сейчас следуют углеродному шовинизму. Судя по всему, только углерод способен образовывать так много соединений, чтобы на его основе сформировалась жизнь. Даже ближайший аналог, кремний, образует гораздо меньше веществ на своей основе. Так что неуглеродные формы жизни сосать, скорее всего.
Аноним 24/09/18 Пнд 14:31:40 #358 №426463 
>>426457
Дай определение жизни в твоём понимании а лучше прочитай нормальные определения, не занимайся словоблудием типа
>все организмы должны биологические быть?
Потому что биология это наука о жизни.
>А зачем тогда этот тред?
Чтобы ты нормально вопросы формулировал, а не нёс пространную хуйню, сам себе отвечая.
другой анон
Аноним 24/09/18 Пнд 14:49:17 #359 №426464 
>>426463
> все организмы должны биологические быть?
Это я не так выразился, потому что вы меня запутали.
> Чтобы ты нормально вопросы формулировал, а не нёс пространную хуйню, сам себе отвечая.
Что я такого странного спросил?
Я задал вопрос "Какие формы жизни могут существовать", тут только конченый не поймёт. Для тебя разжую, то есть, если на земле основа жизни - вода, то какова с научной точки зрения существования жизни на основе кремния, метана и т.д. То есть не только на основе воды, а газа или химического элемента какого нибудь. То есть, высока ли вероятность с научной точки зрения существования подобных форм жизни. Блять, что тут странного, тупого, нелогичного и ненаучного?
Пообщаясь с вами я и себя уже не таким тупым теперь считаю.

>>426460
Ну вот же, он понял и вполне понятно объяснил.
Аноним 24/09/18 Пнд 14:56:02 #360 №426466 
15374700847590.jpg
Так ЕмДриве оказался работающей штукой?
Аноним 24/09/18 Пнд 15:03:23 #361 №426468 
>>426466
Нет, конечно (в смысле, как безопорная хуйня).
Аноним 24/09/18 Пнд 18:31:36 #362 №426518 
>>426468
Так а на что он опирается? Магнитное поле Земли?
Аноним 24/09/18 Пнд 22:12:10 #363 №426556 
>>426518
На собственные электрические кабели вроде.
Аноним 24/09/18 Пнд 22:19:48 #364 №426557 
>>426464
Открой на википедии статью типа «алтьтернативная биохимия». Все. То, что ты ищешь - там.
Аноним 24/09/18 Пнд 22:36:57 #365 №426569 
Петросян1340660157822.jpg
>>426518
На пол гаража, в котором его испытывают.
Аноним 24/09/18 Пнд 23:30:11 #366 №426581 
Последние дни в поддсье можно было наблюдать желто-оранжевую звезду правее и ниже луны. Это Марс? Я до этого даже как-то и не думал, что другая планета может быть такой же яркой, как самые яркие звезды.
Аноним 24/09/18 Пнд 23:35:27 #367 №426583 
>>426581
Скорее всего Марс. Можешь качнуть Stellarium и убедиться.
Аноним 25/09/18 Втр 00:29:43 #368 №426588 
Вопрос про электромагнитное излучение.
Представим себе 2д портал в виде дырки. Из одного конца вселенной в другой конец. Действует он по аналогии с отверстием в гипотетическом непроницаемом барьере, поглащающие все волны и излучения его коснувшиеся.

Будут ли проникать волны больше или меньше, чем отверстие? А можно ли как-то сдетектировать непроходимые волны?
Аноним 25/09/18 Втр 06:03:17 #369 №426603 
Позавчера в ДС была странная луна с "аурой" (как ее мамка назвала) или как я ее назвал "гало". Это какое-то йоба явление или локальная хрень из-за погоды? Впервые видел.
Аноним 25/09/18 Втр 06:11:33 #370 №426605 
>>426603
Тоже видел, хотя я даже не из России.
Аноним 25/09/18 Втр 07:10:11 #371 №426611 
Кто нибудь слышал про этого человека - "Норман Бергран", в контексте инжиниринга НАСА?
Аноним 25/09/18 Втр 07:49:08 #372 №426616 
>>421803 (OP)
Что такое приливные силы? Чего они все разрушают?
Аноним 25/09/18 Втр 08:24:16 #373 №426618 
>>426616
Гравитация убывает с квадратом расстояния. Таким образом, если тело достаточно большое, то та его часть, которая ближе к массивному телу будет притягиваться к этому массивному телу сильнее своей дальней части и поэтому тело будет испытывать разные неприятные нагрузки на разрыв.
Аноним 25/09/18 Втр 10:08:50 #374 №426622 
>>426611
>Норман Бергран

https://spanglermortuary.com/tribute/details/1033/Norman-Bergrun/obituary.html
Аноним 25/09/18 Втр 12:23:57 #375 №426631 
>>426618
Примерно как желе на краю тарелки изгибается и может порваться?
Аноним 26/09/18 Срд 00:38:16 #376 №426723 
>>421803 (OP)
Сап спейсач. Я долбаеб, запсотил вопрос в загибающийся тред, дублирую сюда.
Вопрос знатокам ото. Извсетно, что вселенная расширяется, скорость зависит от расстояния вроде 200км/с на миллион парсек. Так что, в те области которые за 14 млрд световых лет хуй попадешь, так как улетют от нас они на сверхсветовых скоростях.
Теперь вовпрос. Положим я сажусь в корабль, разгоняюсь до 0,(9) с. Вселенная для меня сжимается вдоль оси направления движения. Те звезды которые были от меня в 14 млрд св. лет до этого, становятся в 14 км. Они так же будут улетать со световой скоростью?
Аноним 26/09/18 Срд 03:58:36 #377 №426737 
Было когда нибудь такое, вот едет какой-нибудь инопланетный трактор по %планетанейм% и проезжает в том же месте где проезжал другой трактор, и все такие: "о а я знаю эту местность"?
Аноним 26/09/18 Срд 04:52:27 #378 №426741 
>>426631
Не совсем. Представь себе унитаз, ты в него отложил здоровенную личинку и нажал на смыв. Унитаз при этом кондовый советский и воду пускает не равномерно по всей площади, а сливает основной объём по центру, то есть мощность потока воды в центре больше, чем по краям. Если рассмотреть только половину унитаза (разрезав его вдоль), то поток воды это гравитация, колбаска это то самое достаточно крупное тело, канализация это массивное тело (центр притяжения). Коричневый дружок начинает испытывать разный напор воды в разных своих участках, и тут вступает в дело сопромат.
Аноним 26/09/18 Срд 05:03:43 #379 №426742 
>>426603
Если просто "аура", то это была корона. В холодную влажную погоду рядовое явление.
Если кроме свечения вокруг была ещё и дуга, или две, или целое кольцо, можно с точками усиления — это уже гало. Оно требует специфической погоды.
Аноним 26/09/18 Срд 09:55:32 #380 №426755 
>>426737
Ты когда с утра свой пост перечитал, сам-то смог разобрать по частям тобою написанное? Даже хуй знает, что на такое можно ответить.
Аноним 26/09/18 Срд 10:00:50 #381 №426756 
spacelold.jpg
>>426741
Обожаю двач. Объяснять приливные силы на примере говна
Аноним 26/09/18 Срд 10:35:08 #382 №426759 
>>426755
Да норм всё.
>>426737
Инопланетные трактора - штука редкая и дорогая, потому, чтобы выжать из них побольше информации, их не запускают в те места, которые до них уже объездили другие трактора. Потому нет.
Аноним 26/09/18 Срд 11:01:07 #383 №426760 
>>426756
Двач и так весь из говна. Оно скрепляет и объединяет нас.
Аноним 26/09/18 Срд 11:53:18 #384 №426761 
>>426723
Хз, но логично что если объект убегает от тебя со скоростью С+ то ты, имя скорость С, никак его не догонишь, как бы вселенная не сжималась
Аноним 26/09/18 Срд 11:54:02 #385 №426762 
>>426737
Американцы привозили на Землю части АМС, которая успела помариноваться пару лет на Луне. Единственное отдаленно подходящее к твоему запросу, кроме фоточек лендеров с орбиты.
Аноним 26/09/18 Срд 11:56:49 #386 №426764 
>>426737
этих тракторов всего пару штук и запускали их в разные места так что нет
Аноним 26/09/18 Срд 12:05:30 #387 №426766 
image.png
>>426764
Вот кстати их перечень и сколько они успели проехать. Оче мало как видишь - ты за день можешь пробежать больше чем марсоход за 10 лет проехал
Аноним 26/09/18 Срд 12:07:21 #388 №426767 
>>426723
>Они так же будут улетать со световой скоростью?
Ну не со световой, а с той, что было, но минус световая. Хотя тут уже от точки отсчета будет зависеть. Вместе со скукожевшейся вселенной, ровно на тот же параметр Лоренц-фактора замедлится время.
Аноним 26/09/18 Срд 12:25:49 #389 №426772 
>>426766
Ого Луноход дал пососать
Аноним 26/09/18 Срд 12:49:29 #390 №426778 
image.png
>>426761
Сибляди в очередной раз соснули у С++-бояр.
Аноним 26/09/18 Срд 12:50:42 #391 №426779 
>>426772
Ну так луноходы были фактически не исследовательскими аппаратами а спортивными, с главной целью именно что дать пососать. Заодно ачивку "первый спорткар в космосе" у Маска спиздили.
Аноним 26/09/18 Срд 13:15:22 #392 №426784 
>>426779
У лунохода до земли пинг две с половиной секунды, им почти в реалтайме можно было управлять, а у марсоходов средний пинг – полчаса, потому и ездят медленно и аккуратно
Аноним 26/09/18 Срд 13:18:19 #393 №426785 
Мне сейчас рассказали байку про космические телескопы. Якобы первый телескоп предложил запустить не учёный, а менеджер, чтобы показать быдлународу что учёные не хуи у себя в космических агентствах пинают.
Насколько это правдиво? А то мне чёт не верится нихуя
Аноним 26/09/18 Срд 13:21:00 #394 №426786 
>>426784
А им и в реалтайме управляли, с таким пингом, и суть подготовки водителей было не "едьте аккуратно, главное не проебать аппарат любой ценой" а "как бы побольше проехать".
Аноним 26/09/18 Срд 14:48:42 #395 №426791 
>>426767
>>426761
Хорошо. Выходит я сижу в корабле, смотрю на звезду А в 14 км и вижу, что нихуя не могу ее догнать. Эта звезда по оси х. Потом я смотрю по перпедикулярной движению оси у, и вижу, что другая звезда В по оси у в 14 млрд световых лет отдаляется с той же скоростью, что и звезда А, в 14 км. Выходит, что в моей с.о. расширение анизатропное, и относительность равномерного двиижения - это все пиздежь?
Аноним 26/09/18 Срд 14:55:40 #396 №426795 
15377097784830.jpg
Вопрос не по теме, но не знаю где еще спросить, а тут всегда помогают с тупыми вопросами: постоянно вижу в фильмах и играх что в древности были йоба конструкции, типа храмов, всякие александрии с колоннами, гигантские сооружения и т.д. Но где они сейчас?
Аноним 26/09/18 Срд 14:58:52 #397 №426798 
luxor-temple-egypt-dc8cf49c7e97.jpg
>>426795
Внезапно, там же, где и до этого.
Аноним 26/09/18 Срд 15:02:27 #398 №426800 
>>426798
Т.е. все эти вавилонские сады, александрийские библиотеки, колоссы и прочее - есть на самом деле?
Аноним 26/09/18 Срд 15:18:44 #399 №426804 
hqdefault.jpg
>>426800
Аноним 26/09/18 Срд 15:20:13 #400 №426805 
1537122190895.jpg
>>426804
Я просто дальше черты своей мухосрани не ездил и думал что это всё легенды, кроме всяких чоень известных разрушенных колизеев или стоунхенджа.
Аноним 26/09/18 Срд 15:28:16 #401 №426806 
zog.gif
>>426805
А может так оно и есть?
Аноним 26/09/18 Срд 16:38:33 #402 №426810 
>>426800
А еще земляшка плоская кста.
Аноним 26/09/18 Срд 20:01:41 #403 №426844 
>>426786
собственно да, луноходу не надо было останавливаться часто и делать много исследований. зажал газ и похуярил. Плюс там гравитация ниже так что бензина меньше расходуется и ветер в морду не дует - вот такими хитростями и перегнали марсоходы
Аноним 26/09/18 Срд 20:41:01 #404 №426866 
>>426844
>луноход
>бензин
Дядя, ты дурак?
Аноним 26/09/18 Срд 20:46:44 #405 №426868 
>>426866
Слушай, а когда мы тут про жидкий вакуум петросяним, ты это тоже буквально воспринимаешь?
Аноним 26/09/18 Срд 20:49:56 #406 №426871 
>>426868
Двачую.
Я про солярку на ракетах шуткую, обычно понимают.
Зачастую в смысле дельты: солярки не хватит 20 тонн на гсо тащить = дельты не хватит.
Аноним 26/09/18 Срд 21:12:29 #407 №426874 
SESpaceTruckin-2.3.jpg
>>426871
Святые яйца фон Брауна, а что если наши местные преколы это на самом деле сложна? Придётся составлять справочник для новоприбывших и мимопроходящих. Ну или банить на входе в раздел азаза
Аноним 26/09/18 Срд 21:14:21 #408 №426876 
>>426874
>банить на входе в раздел азаза
Пробовали.
Полная хуйня получается.
Аноним 26/09/18 Срд 21:20:44 #409 №426879 
>>426876
Это когда такое на спейсаче пробовали и в чём тогда было отличие от унылофорумов?
Аноним 26/09/18 Срд 21:42:41 #410 №426896 
>>422149
На самом деле это не квазар, а инопланетный крейсер. Они летели сюда, чтобы вступить в контакт, но когда подошли поближе и рассмотрели нас, почему то развернулись и начали съебывать с постоянным ускорением. На фото реактивные струи ионного двигателя.
Аноним 26/09/18 Срд 21:50:54 #411 №426900 
>>425154
1) Абсолютно неверно.
2) Абсолютно верно.
Аноним 26/09/18 Срд 21:53:45 #412 №426903 
>>425261
Теоретически, в ведре говна столько потенциальной энергии, что весь мир можно месяц электроэнергией обеспечивать (на максимальной нагрузке).
Аноним 26/09/18 Срд 22:01:00 #413 №426906 
>>426900
>Абсолютно неверно.
Инфа сотка?
Аноним 26/09/18 Срд 22:01:26 #414 №426907 
>>426906
Да.
Аноним 26/09/18 Срд 22:24:59 #415 №426917 
А правда что ариан значит ариец?
Аноним 26/09/18 Срд 22:27:25 #416 №426918 
>>426917
Arian - да, ариец/арийский.
А вот у ракеты Ariane название переводится как Ариадна.
Аноним 27/09/18 Чтв 02:02:16 #417 №426951 
Расскажите что за астрофорумы вы всё время здесь упоминаете, есть кулстори или не более чем спейсач с авторизацией?
Аноним 27/09/18 Чтв 09:39:55 #418 №426984 
>>426951
Есть форум бесславно почившего в бозе научпок-журнала Звездочет. Раньше там было дохуя годных авторов, следивших за конкретными темами, тот же Сергей Попов (астрофизик-профессионал, регулярно делавший обзоры свежих статей по астрофизике). Со временем форум скатился в говно и был оккупирован школотой, сайнс-фриками и шизоидами напотипу антисетипетуха.

https://astronomy.ru/forum/

Еще пока есть форум Новостей Космонавтики (еще один рипнувшийся журнал, но уже с более серьезными авторами). Там сохранился своеобразный олдфажный контингент со своими традициями и мемасиками НННШ. Отдельный плюс - инсайдыри из ЕКА и разных конторок вроде Лавочки. Охуенный минус - сервак форума полумертвый, и при любом шухере падает.

http://novosti-kosmonavtiki.ru/forum/
Аноним 27/09/18 Чтв 09:43:47 #419 №426985 
>>426791
>другая звезда В по оси у в 14 млрд световых лет отдаляется с той же скоростью, что и звезда А, в 14 км
Бля, сорре, очень лениво думать как объяснить, больно глубоко копаешь. Но я выделил тебе изъян в рассуждениях. Можешь проработать сам, а можешь счесть меня тупым валенком, зомбированным для защиты картины мира, выстроенной нам жидами-рептилоидами. Сам реши кароч, как-то похуй ваще.
Аноним 27/09/18 Чтв 14:50:20 #420 №427047 
>Вокруг Марса вращается два небольших спутника неправильной формы – Фобос и Деймос диаметром в 22 и 12 километров, удаленные от поверхности красной планеты всего на десять и 23 тысячи километров. Многие ученые считают, что это не планеты, а астероиды, захваченные притяжением Марса в далеком прошлом.

в чем разница? Чем астероид отличается от карликовой планеты?
Аноним 27/09/18 Чтв 15:45:38 #421 №427059 
Сколько энергии в тротиловом эквиваленте у ста грамм фотонов?
Аноним 27/09/18 Чтв 15:46:01 #422 №427060 
image.png
>>427059
Картинка отклеилась
Аноним 27/09/18 Чтв 15:59:27 #423 №427065 
>>427060
У фотонов в привычном понимании нет массы, ну да похуй. E=mc2=0.1331016=91015 Дж
1 мегатонна (Мт) ТНТ = 4,184·1015 Дж
Итого 2 мегатонны с копейками. Начинать разговор за некорректность такого расчета не буду. Он некорректен ровно настолько, насколько некорректен вопрос. Так что смею предположить, что тебя такое должно устроить. Мог бы и сам посчитать, ведь не ментальный инвалид, наверное.
Аноним 27/09/18 Чтв 16:01:02 #424 №427067 
>>427065
Блять, заморочился со степенями, но забыл про звездочки, сук.
>E=mc2=0.1x3x3x1016=9x1015 Дж
Аноним 27/09/18 Чтв 17:49:19 #425 №427122 
анон подскажи в какой из тредов зайти чтобы почитать что нить про японский зонд который видео сделал из снимков пребывая на астеройде
новость тут увидел
https://www.metronews.ru/novosti/world/reviews/zond-snyal-pervoe-v-istorii-video-s-poverhnosti-asteroida-1461442/
Аноним 27/09/18 Чтв 17:50:42 #426 №427123 
>>427122
>>398001 (OP)
Аноним 27/09/18 Чтв 17:51:02 #427 №427124 
>>427122
>>398001 (OP)
>>427028
Аноним 27/09/18 Чтв 17:56:32 #428 №427127 
>>427123
>>427124
благодарю
Аноним 27/09/18 Чтв 21:24:19 #429 №427148 
дети против волшебников.jpg
>>427047
В форме, а следствие в размере. Если астероид круглый как Церера, то он уже не астероид, а карликовая планета. Для этого достаточно 400-600 км радиус, в зависимости от материала. Но эта иерархия применима только к вращающимся вокруг солнца. Спутники же все едины. Что ганимед, что мелкая шелупонь за 3 км, всё считается спутниками.
В твоём тексте скорее всего имелось ввиду не "что это не планеты, а астероиды", а "это не спутники, что были изначально, а захваченные астероиды".

По сути это всего лишь вопросы терминологии, можешь хоть карасиком назвать. Определились пока с определением планет, карликовых планет и астероидов. Да и то астероид и метиороид, довольно расплывчато определяют. По сути всё камни, но человекам надо всё систематизировать, отсюда и батхёрт.
Аноним 28/09/18 Птн 01:18:31 #430 №427168 
>>426985
Неа, не понял нихуя. Понятие скорости вполне определено в системах отсчета. Например Луна из-за расшиерения пространства отдаляется на пару нанометров в год, гипотетически это измеримо. Непрерывная дифференциируемая величина, которая своей производной и дает понятие скорости.
Или на что ты намекаешь?
Аноним 28/09/18 Птн 05:51:05 #431 №427180 
>>427168
>Например Луна из-за расширения пространства отдаляется на пару нанометров в год, гипотетически это измеримо
Ой бля, нат зис щит эгейн. Представь резиновый шариксука бля, анозер щит эгейн. В нем проделана куча отверстий-дырочек обычной иголкой. И вот ты прижимаешь шарик к клапану баллона с гелием. Газ наполняет шарик, но уходит через расширившиеся под давлением дырочки. Сила упругости материала не позволяет шарику разлетаться на куски и в какой-то момент, если поток газа из баллона будет идти с постоянным давлением. Шарик достигнет равновесия формы, а газ так и будет вылетать сквозь дырки. Силой упругости для материи на космических масштабах выступает гравитация. Увеличь постоянную Хаббла - увеличится длина "поводка" или объемы ограниченного телом шарика пространства. Но силу никто не отменял. Луна удаляется по херовой туче причин, описанных небесной механикой, но не по причине расширения пространства. Его влияние при нынешнем показателе становится заметным только на галактических масштабах, внутри галактик объекты достаточно гравитационно связаны, чтобы удерживать равновесие. Когда давление увеличится, тогда шарик лопнет, не раньше. И еще у тебя проеб в расчетах. Между Луной и Землей в линейной проекции размерность ежесекундно по показателю константы увеличивается на 0,8 нм. За год там нарастает целый дюйм. Но даже такую дельту параметра хер измеришь с учетом перигей-апогей эксцентриситета и громоздкости формул механики для прецизионных вычислений. Чем точнее - тем длиннее интегралы.
Но к примеру между Юпитером и Солнцем за миллиард лет наприрастало 85 млн км. Этот параметр был бы уже заметен опять же из выводов небесной механики, ибо очень массивен, и такие изменения сильно повлияли бы на стабильность системы. Это мы вычислить способны. И ничто не говорит о том, что его орбита настолько менялась. Собственно ничто пока не говорит о дискретности расширения пространства, поэтому вполне очевидна, что коррекция гравитацией происходит в прямом эфире, а все избытки пространства убегают в пустоты меж гравитационно связанными комками материи.
Аноним 28/09/18 Птн 08:56:43 #432 №427188 
>>427180
Ну насколько я знаю, ОТО вообще механика бессиловая.
С луной да согласен, есть факторы поболее. Но если представить систему из 2 изолированных мат. точек, с массой, где одна точка на идеальной круговой орбите другой точки, то расширение вселенной будет единственным фактором, повышающим орбиту, гравитация тут уже ниче не удержит.
В общем хуй с ним со всем этим, единственное что меня волнует - верно ли утверждение, что даже на релятивистских скоростях, сжатие вселенной не дает добраться до далеких звезд. И если это так, то что делать с анизатропностью расширения и относительностью движения. Меня анон тут подталкивает, но я нихуя не могу допедрить. Аналогия с шариком пока нихуя не помогает, так как не понятно, как там скорости смотреть. Вангую, что скорость звука в резине принимать за "скорость света"
Может еще какие-то проблемы существуют, с замедлением времени, я хуй знает, но в моем представлении в движущейся с.о. померить расстояние до звезды не проблема.
Короче я еще подумаю, тоже подключайтесь, хочется понять.
Я недавно читал статейку, про акустические модели СТО DOI 10.1007/s10701-017-0109-0, но с ОТО все сложнее, для нее вроде аналоговой модели еще не построено.
Аноним 28/09/18 Птн 13:37:51 #433 №427241 
>>427188
>Но если представить систему из 2 изолированных мат. точек, с массой, где одна точка на идеальной круговой орбите другой точки, то расширение вселенной будет единственным фактором, повышающим орбиту, гравитация тут уже ниче не удержит.
Как раз система с 2 объектами, где есть гравитация, и есть эта сила. Идеальной круговой не будет, рано или поздно она перекосится ибо гравитационные волны.

мимо
Аноним 28/09/18 Птн 23:05:09 #434 №427454 
На фотон воздействует гравитация. Но порождает ли фотон гравитацию?


Когда уже померят полную скорость Андромеды?
Аноним 29/09/18 Суб 01:02:04 #435 №427473 
Что это такое у HTV на втором пике >>427444 вокруг захвата для манипулятора? Это отсек с негерметичным грузом? Для чего там поручни под человеческие руки установлены?
Аноним 29/09/18 Суб 01:03:05 #436 №427475 
А бля, сообразил. Там два захвата, на пике для захвата не корабля, а груза.
Аноним 29/09/18 Суб 01:07:18 #437 №427476 
>>427475
Ебать ты хитрый, сам спросил, сам ответил.
Аноним 29/09/18 Суб 02:12:38 #438 №427477 
>>426723
Вселенная не может расширяться, т.к. она бесконечна.
Аноним 29/09/18 Суб 02:38:27 #439 №427479 
image.png
>>427454
>На фотон воздействует гравитация. Но порождает ли фотон гравитацию?
Конечно. Вот же формула.
Аноним 29/09/18 Суб 13:12:44 #440 №427523 
>>427477
Гениально
Аноним 29/09/18 Суб 13:13:47 #441 №427524 
>>427477
>она бесконечна
Сие заявление было бы неплохо подтвердить.
Аноним 29/09/18 Суб 15:17:39 #442 №427537 
Есть ли какие-то изыскания по поводу формы магнитосферы звёзд на разных этапах эволюции? А во внегалактическом пространстве?
Аноним 29/09/18 Суб 15:29:55 #443 №427540 
Не понимаю.

Есть боинг 777 в варианте 300ER. Берет на борт чуть меньше 400 пассажиров. Стоит эта няша около 320-300 млн долларов. Первая ссылка в поисковике говорит, что семь крупнейших компаний в штатах имеют ~9% маржу с полета и порядка 20 баксов прибыли с пассажироместа. Это средняя по больнице на все рейсы как международные, так и местные. С кучей нюансов.

За один рейс 777 компания делает 20 х 400= 8000$ прибыли. Предположим, борт летает 24/7, средний рейс с посадкой/высадкой часа 4. Длиннее маршрут - больше прибыли, но меньше рейсов. И наоборот. Считаем среднее. 6 рейсов в день. 6 х 8000 = 48000$ в день. Считаем, что борт летает 365 дней в году. 365 х 48000 = 17.5 млн $. Т.е. чтобы просто окупить лизингсебя 300ER должен летать 20 лет. Но, борта не летают на 100% заполненные. Пассажиропоток днем больше, чем ночью. Самолет должен проходить регулярное ТО. Мелкий и капитальный ремонт. Прочие межполетные процедуры. Самолет может простаивать по тем или иным другим причинам. Забастовки. И т.д. Я могу предположить, что там не 20 баксов, а 30 дерут с места. Или даже 40. Но всё-равно порядок цифр не меняется.

Что это за авиабизнес такой????? С отсрочкой прибыльности в -нцать лет и 9% маржей? Это вообще нормально? Зачем нужны такие дорогие самолеты? Кто их покупает?
Аноним 29/09/18 Суб 15:41:28 #444 №427546 
>>427540
Забыл отдельную плату за багаж, миллиард всевозможных комиссий, существование классов помимо эконома, платные опции типа выбора места и торговлю всякой хуйней на борту (вроде интернета по безумной цене). Вполне нормальная ситуация, что самими билетами торгуют едва ли не по себестоимости (а на некоторых рейсах и ниже нее), а всю прибыль генерируют платные опции, так много индустрий работают.

Самолеты, кстати, не покупают, а берут в долгосрочный лизинг обычно, потому что единовременно выложить миллиарды долларов не может практически никакая авиакомпания, и производители входят в положение.
Аноним 29/09/18 Суб 15:42:17 #445 №427547 
>>427540
Так-то авиокампания самолёт ещё должна купить у авио-строителя. А авиокампания продаёт билеты куда больше чем 20 долларов.
Аноним 29/09/18 Суб 15:43:28 #446 №427548 
>>427546
>долгосрочный лизинг
То бишь в кредит. И помимо маржи, ещё и за сам самолёт берут.
Аноним 29/09/18 Суб 16:08:10 #447 №427567 
А все просто. Ты путаешь доход и прибыль. 20 долларов прибыли образуются уже после всех расходов. Если ты конечно привел правильные цифры.
Аноним 29/09/18 Суб 16:28:38 #448 №427573 
Да нет, цифры правильные. С условного билета за 355 баксов компания получает 17 долларов прибыли. Но это, действительно, без учета багажа и прочего. Ебанутый автор, перепостил сферическую хуйню в вакууме из заголовка, а потом основываясь на ней пост хуячит. Вот этого я не видел, думал с багажем как минимум, ну и что места в бизнес-классе погоды не делают:

>In truth, airlines now cover their costs with tickets and get their profits from baggage fees, seat fees, reservation-change fees and just about all the other nickel-and-diming that aggravates customers. U.S. airlines were on pace to take in more than $4 billion in baggage fees and $3 billion in reservation-change and cancellation penalties in 2017, according to Transportation Department data. (The full year hasn’t been tallied yet.) The two categories add up to about more than half of the net profits airlines posted last year

Аноним 29/09/18 Суб 16:38:36 #449 №427575 
Так в любом случае, в авиакомпании на калькуляции не птушник сидит уж всяко. И net profits считает уже после вычета налогов даже. Так что все лизинговые и прочие издержки уже учтены.
Аноним 29/09/18 Суб 18:08:31 #450 №427587 
>>427537
Есть.
Аноним 29/09/18 Суб 18:14:04 #451 №427590 
>последние потсы про авиацию
И кто-то ещё смел кукарекать что огурцам не место в спейсаче из-за авиации.
Кстати да - че тут забыли-то? К космосу авиация не особо.
Аноним 29/09/18 Суб 18:29:48 #452 №427595 
>>427590
Тащем-то космос вырос из авиации.
Аноним 29/09/18 Суб 18:39:06 #453 №427603 
>>427595
[покачивая ладонью]
Ну такое. Немножко. Реактивное движение подтолкнуло обе отрасли и оно было отдельно во всех странах.
Аноним 29/09/18 Суб 18:53:56 #454 №427609 
>>427603
Всего два слова, реактивный ты наш.
АВИАЦИОННО-КОСМИЧЕСКАЯ ПРОМЫШЛЕННОСТЬ
Аноним 29/09/18 Суб 19:00:04 #455 №427613 
>>427609
Йоу. Я ж не спорю.
Только ИРЛ это две разные вещи.
Орбитальная механика и воздушный полёт - это как полёт на аэростате и покатушки на подлодке.
Разные.
Вещи.
То, что они сверху - это так решили их объединить. И в целом верно, летуны на истребителях более способны в удержание блевотного рефлекса от недогрузок.
Но в этом общность кончается.
sage[mailto:sage] Аноним 29/09/18 Суб 19:04:59 #456 №427614 
>>427613
>полёт на аэростате и покатушки на подлодке
Даже в этом больше общего.
Аноним 29/09/18 Суб 19:06:19 #457 №427615 
>>427614
Опять покачну ладонью.
Ну примерно да.
Аноним 29/09/18 Суб 19:06:49 #458 №427617 
>>427613
Ты узко видишь только на сам полет.
Как минимум забываешь про металлургию легких сплавов, мех. обработку металлов, аэродинамику, системы ориентирования и связи.
Аноним 29/09/18 Суб 20:09:35 #459 №427642 
>>426868
Конечно! А что, не надо было?
Аноним 29/09/18 Суб 21:41:22 #460 №427648 
На Юпитере наверно ебать какой грохот стоит из-за тамошних ветров? Тама 500 децибел ебашит думаю или около того
Аноним 29/09/18 Суб 22:03:49 #461 №427654 
SPCJournal2.jpg
Не знаю куда вбросить, пусть будет в главном треде борды.
Аноним 29/09/18 Суб 22:08:11 #462 №427656 
15371251064050.jpg
>>427654
Антош, ты себя береги, а то не ровен час выдохнешься нахуй.
Лучше подсоби как ты (вроде ты же, да?) запилил смеющуюся пикчу про космонавтов, но про огурцов.
Аноним 29/09/18 Суб 22:09:07 #463 №427657 
15361778756330.jpg
image.png
>>427656
>смеющуюся пикчу про космонавтов, но про огурцов.
1 - вроде ты.
2 - Я в процессе херачения.
Аноним 29/09/18 Суб 22:27:15 #464 №427658 
https://youtu.be/_DnDeBa0KFc
А че выходит, все просмотры в одну сторону галактики идут что ли? Понятно что вне плоскости особо не стоит смотреть, но дальше по диску почему не равномерно?
Аноним 29/09/18 Суб 22:38:38 #465 №427659 
>>427656
>Пик
А есть какой-то отдельный тред с этим? А то я в топ добавил себе ттв и телескопотред, а на сам /spc не захожу почти. Вон, даже не знал что огурцов вернули
Аноним 29/09/18 Суб 22:40:52 #466 №427660 
>>427659
Никак нет. Слоудоска, особо тредов не позапилишь.
Аноним 29/09/18 Суб 23:05:36 #467 №427665 
NSJJournal.jpg
>>427656
>>427657
>Антош, ты себя береги, а то не ровен час выдохнешься нахуй.
Рано или поздно всё равно выдохнусь - а сейчас пилю так много, потому что доставляет.
>Лучше подсоби как ты (вроде ты же, да?) запилил смеющуюся пикчу про космонавтов, но про огурцов.
Да, мой покак. Чем подсобить-то, скафандры подрисовать?
>>427659
>Вон, даже не знал что огурцов вернули
О, журнал теперь даже на самом деле выполняет новостную функцию.
>>427660
У нас раньше кто-то запилил "тред творчества анона", где мы складывали весь свой кактент, но его смыло дискорд-вайпом, весной вроде а ещё смыло мой космофактов тред, до сих пор жопа горит. Можно возродить в принципе.
Аноним 29/09/18 Суб 23:08:18 #468 №427666 
>>427665
>Да, мой покак. Чем подсобить-то, скафандры подрисовать?
Огуречно допилить. Чтоб зеленые оба три были и сзади еще чето взрывалось, как полагается огурцам.

>Можно возродить в принципе.
Возрождай, вещь хорошая.
Аноним 29/09/18 Суб 23:38:44 #469 №427672 
15382481482971.jpg
>>427666
ХАЙ КВОЛИТИ ЩЕТ, БОЙЗ
Пойдёт?
Аноним 29/09/18 Суб 23:40:47 #470 №427673 
>>427672
Содомит!
Мой самый любимый содомит на доске.
Я потом по-своему доделаю, но ты как обычно охуенен и радуешь.
Желаю тебе... бля, обычно рак желаю. Что противоположно раку? Вот это короче желаю. Живи и радуй нас дальше.
Аноним 29/09/18 Суб 23:45:06 #471 №427676 
>>427665
>У нас раньше кто-то запилил "тред творчества анона
Я запилил.
Вкатывайся, выкладывай свои шедевры по очереди.
>>427674 (OP)
Аноним 29/09/18 Суб 23:48:18 #472 №427679 
>>427673
:3
>>427676
Ок. Я и ваши шедевры повыкладываю - у меня всё схоронено.
Аноним 30/09/18 Вск 01:15:52 #473 №427732 
zPxMJR.jpg
Так-то авиация тоже использует орбитальную механику. И в перспективе вся авиация это сугубо суборбита. Или станете переобуватся только с появлением данного явления. Дескать это не космос, а теперь то же явление космос. Вон там какая-то утка была с ёба суборбитером от МО РФ. По сути это Я хз правда ли это, допустим правда, да и теоретически это возможно, самолёт-суборбитер. И уже нельзя разделить авионику и космонавтику, как разные явления. Так что полёты в тропосфере ничем не отличаются от межпланетарных.
Имхо
sage[mailto:sage] Аноним 30/09/18 Вск 01:43:08 #474 №427736 
хн.jpg
>>427732
Аноним 30/09/18 Вск 01:52:03 #475 №427738 
dont make me mcslap you.jpg
>>427732
Извини, братюнь, херь полнейшая что ты написал.
Аноним 30/09/18 Вск 02:13:49 #476 №427743 
>>427736
>>427738
Обоснуйте. Согласен что "ничем не отличаются" жудкое утрирование, на данный момент. Но тем не менее, одно вытекает из другого и в перспективе сливается в неотличимое друг от друга понятие. И моя претензия в основном в том что нельзя переобуватся, что сейчас нет, а потом да.
sage[mailto:sage] Аноним 30/09/18 Вск 05:17:13 #477 №427759 
>>427648
Ты, видимо, не в курсе, что децибел, как собственно и бел - величина логарифмическая. Поэтому разница между 150 дБ и 500 не в 3.33 раза громче, а в 1035 выше энергия звуковых волн. Надеюсь теперь ты можешь сделать вывод, что таких энергий в веществе в привычных агрегатных состояниях быть не может.
Аноним 30/09/18 Вск 09:31:17 #478 №427766 
Specific-impulse-kk-20090105.png
>>427743
Если использовать ракетные движки, то неэффективно по топливу. Боинг 767, например, жрет менее 5 тонн топлива в час и несет ~72 тонны топлива. При стартовой массе в 180 тонн и УИ 350с это даст ~1.8 км/c скорости. При УИ 450с (ага, с понтовыми водородниками закрытого цикла, причем стартуем в вакууме) будет 2,3км/c. Это ниже, чем у Фау-2, с дальностью ажно 320 км (для сравнения у Боинга дальность 9400 и более, в зависимости от модификации).
А ВРД, способные вывести самолет на суборбиталку пока даже вояки не заполучили, а когда заполучат будут едва тянуть по финансам.
Плюс, гиперзвуковые ВРД будут все равно менее эффективны, чем дозвуковые турбовентиляторники. Пикрелейтед как УИ падает с ростом эффективной скорости.
Аноним 30/09/18 Вск 12:25:21 #479 №427838 
>>427759
Хорошо, но там слышно же грохот урагана на Юпитере? Может какой-нибудь зонд записал шум?
Аноним 30/09/18 Вск 12:39:51 #480 №427849 
>>427838
Этот шум можно смоделировать на основе наблюдательных данных. Но обычно тебе подобные говорят, что так не считается. Но чтобы записать звук, нужно звуковые колебания поймать будучи в среде распространения. Космический аппарат имени покровителя этого треда был пущен в атмосферу Сатурна и его закономерно raspeedoraselo. В случае с Юпитером все будет еще сложней, потому что объект массивней, орбитальная скорость выше. Прямой наводкой тоже не выйдет, те же 10+км/с относительной скорости, как не пытайся. Запустить огромную хуиту с баками и движком, который погасит такую скорость, сейчас нереально. Сорре, братан, фундаментальная наука она не про "ебать наверное горячо в центре звезды".
Аноним 30/09/18 Вск 13:16:17 #481 №427856 
15085089572740.jpg
WB GDP ranking.jpg
Простите за офтоп, не в порашу же мне идти с такими вопросами. Только на /спейс сидят взрослые и серьезные анонимусы, которые по хардкору пояснят за любое явление во вселенной и сумеют сдержаться от скатывание треда в хохлосрач и поиски оных под кроватью.

Есть две боевые картиночки, одна из них в народе называется "2% дерьма мировой экономики", а другая с википедии на ту же тему. Получается, картинки врут? Россия - вовсе не 2% мировой экономики, а 6-ая экономика мира и стронк? Да, экономика на сырье, но все же.

Аноним 30/09/18 Вск 13:32:13 #482 №427864 
>>427856
Зашел с /ро-раши значит я такой, пробздетбся в космос. А тут как дома.

Ладно, а теперь вопрос к знатокам. Тот лысый кабан который летает на мкс и выкладывает видосики на ютубе, снял туториал как они на станции ходят по маленькому и по большому. И судя по тому как он руками берет пылесос, который предназначен для смыва, делает он это неуверенно. Смотрит на инструкцию. Не похоже что они там гадят в этот говносос. Так как же все таки справляют нужду? В перерывах между съемками на киностудии?
Аноним 30/09/18 Вск 13:42:30 #483 №427870 
>>427856
На первый очевидно номинальный ВВП в количественном выражении грязной вонючей бумажкой. На втором очевидно список по ППП(паритет покупательной способности), выраженный в интернациональном искусственном манядолларе. Впрочем именно показатель по ППП более показателен. Потому что раздутый ВВП США, также очевидно, раздут именно из-за контроля над мировыми финансовыми институтами - ВБ и МВФ. И еще у них биржа 120% от ВВП, что по сути просто рынок воздуха. Не знаю входит ли капитализация национальной биржи в ВВП.
Хотя все это не отменяет факта, что США стронг, а раша даже по ППП лишь 3% от мировой экономики, пусть и шестое место.
Аноним 30/09/18 Вск 14:32:59 #484 №427882 
>>427849
В Юпитер пускали спускаемый зонд, мань, правда звук он не записывал. Если не разбираешься – не надо с таким охуенным высокомерием слать нахуй вопрошающих
Аноним 30/09/18 Вск 15:24:17 #485 №427908 
sat.jpg
212409883.jpg
Хех, мда. Я думал, кубсат самый маленький формат для спутников. А оказывается есть еще покеткуб (P) с ребром в 5 сантиметров, по объему получается 1/8 1U, со стандартным весом в 125-250 грамм.

https://en.wikipedia.org/wiki/PocketQube
Аноним 30/09/18 Вск 16:29:56 #486 №427930 
>>427766
>Дорохо
>неэкономично
Зато быстро. Прилететь из Москвы в сидней за 10 минут. Дорого но быстро. Быстрее только кротовые норы. К тому же я говорю в перспективе. Т.е в будущем, когда проблемы технологий решатся. И это не отменяет моего тезиса
Будут лёгкие самолёты, чуть потяжелее уже полуракета, а большая уже ёба девайс. Не стоит забывать, что горизонтальный взлёт экономичнее вертикального.
>Ракетные движки.
Зачем? В атмосфере летишь на атмосферных, когда вылетаешь в мезосферу врубаешь ракетные, или не врубаешь и падаешь по суборбите.

Если никакого запрета на промежуточные звенья между ракетой и самолётом нет, то и выделять одно из другого глупо.
Аноним 30/09/18 Вск 17:11:46 #487 №427936 
>>427930
>Зато быстро.
>Прилететь из Москвы в сидней за 10 минут.
Во первых, даже с первой космической, от Москвы до Сиднея пол часа. Но рассчитывать на такое не стоит (я уверен, что ты это проигнорируешь и напишешь, что пол часа тоже неплохо), потому что для такой дальности нужна уже практически орбитальная скорость, около 7 км/c. Для 5500км требуется около 6 км/c. И плуять придется полноценный космический корабль, с теплозащитой, перегрузками при посадке и прочим.
>>427930
>Зачем? В атмосфере летишь на атмосферных, когда вылетаешь в мезосферу
Ты в KSP переиграл. Типичный эффективный самолетный двигатель имеет потолок около 10 км и предельную скорость 0,9 мах. При потребной скорости 18-20 мах. Типичный двигатель, который хоть как-то позволит сделать описываемое тобой, ставится на боевые самолеты с 7-8 значным ценником, конской стоимостью летного часа. и весьма скромной топливной эффективностью. Осбоенно если он умеет стартовать с Земли.
А гиперзвуковых ВРД пока вообще толком нет.
Аноним 30/09/18 Вск 17:12:45 #488 №427937 
>>427936
>А гиперзвуковых ВРД пока вообще толком нет.
Прямоточники жи.
Аноним 30/09/18 Вск 17:14:55 #489 №427938 
гайз, не знаю сюда мне или нет, но может поможете. Сам я гуманитарий, дуб дубом, но вот делаю приятелю крафтовый ящик в подарок, хочу на нём пару надписей изобразить, отсюда вопрос что написать? Про него:-гик 1-2 степени. закончил математическую школу, обожает комиксы про научную фантастику( смотрит батл стар галактика, и тому подобное, шелдонов всяких, потом читает хоккинга и главное понимает, также киберпанк всякий, уважает научный атеизм) Может подскажете что изобразить написать на ящике из дерева или нарисовать ? спасибо, хотелось бы что нить оригенальнее E=mc в степени
Аноним 30/09/18 Вск 17:16:23 #490 №427939 
image.png
>>427938
Полную версию формулы.
Аноним 30/09/18 Вск 17:22:26 #491 №427942 
>>427930
И еще, тут такое есть. Если такой "лайнер" с дальностью 5500км будет весить тонн 30, то запускалка для него сможет элементарно и без напряга быть орбитальной ракетой на 15 тонн полезной нагрузки, исползуя весьма скромный и ненапряжный разгонник, с уи ~330с в вакууме и массовым совершенством ~1:6.
Но тут важно обратное - запуск подобного лайнера будет стоить сравнимо с запуском средней орбитальной ракеты. Так что отстаивай не суборбитальные лайнеры, а копеечную стоимость космических запусков вообще.
Аноним 30/09/18 Вск 17:25:55 #492 №427944 
>>427937
Прямоточники со сверхзвуковым горением, пригодные для скоростей >5 махов, существуют только в виде манямакетов и парочки тухлых экспериментов, до самолетов на подобных двигателях еще десятки лет.
Аноним 30/09/18 Вск 17:31:35 #493 №427947 
>>427937
Ога. Которые пока только в составе летающих лабораторий, для которых успехрекорд это "проработал 5 минут", и которые перед стартом требуется отдельным ускорителем разгонять, потому что работают они только на гиперзвуке.
Аноним 30/09/18 Вск 18:31:37 #494 №427955 
Я полагаю, что женщин пускают в космос, не потому что они такие умные и выносливые. Поэтому мой вопрос примерно в следующем: проводят ли в космосе всякие тайные исследования, в т.ч. по части половой ебли и репродуктивной функции людей?
Аноним 30/09/18 Вск 18:33:07 #495 №427957 
>>427955
Нет, это кококо-негуманно.
Если бы Рейх не выпилили, то они бы экспериментировали.
Аноним 30/09/18 Вск 18:36:56 #496 №427959 
>>427957
Людей заставляют дрочить в пробирку и засовывают шланги в горло/срачло в рамках рутинных мед. процедур, не говоря об испытаниях разных экспериментальных лекарств и методов лечения, граничащих с летальным исходом — а тут негуманно ебаться? Ну ёбаный рот.
Аноним 30/09/18 Вск 18:38:46 #497 №427961 
>>427959
А ну личинус зародится, а ну как из-за невесомости он не сможет в земляшку, так и обречешь ЧИЛАВЕКА на мучительное существование! НИЛЬЗЯ ТАК ЭТАЖИРИБЕНОК!
Фошисты бы по результатам эксперимента сожгли бы нахуй личинуса с мамкой.
Аноним 30/09/18 Вск 18:41:45 #498 №427965 
>>427961
А могли бы сейчас быть сверхлюдьми вместо того, чтобы прокачивать трансгуманизм через смартфоны с картинками котиков в два тапа. Это уже тянет на оправдание фашизма?
Аноним 30/09/18 Вск 18:42:27 #499 №427966 
>>427965
Да. Зарепортил и написал в МВД, жди гостей.
Аноним 30/09/18 Вск 18:48:36 #500 №427969 
1354189209377.jpg
>>427966
Аноним 30/09/18 Вск 20:31:29 #501 №428022 
>>427955
Тайные на то и тайные - потому что никто о них не знает.
Аноним 01/10/18 Пнд 03:19:00 #502 №428062 
Вот я сейчас хуйню что скажу, охуеете.

Вот предположим, что время, как идругие измерения, можно описать очередным взаимодействием. Дескать электромагнитизм ничем не отличается от пространства-время, и оно(время) точно так же распространяется со скоростью света(если можно время через время измерить).
То есть летит свет до нас от какого-то взрыва, скажем ЯВ на луне. В за секунду, до того как там должен прогреметь взрыв посылаем сигнал, он летит секунду. Прогремел взрыв, но для нас это ещё будущее через секунду, в то время как на луне настоящее. И когда говорят: "вот там что-то происходит только свет не долетел до нас", не верно, но верно: "там что-то будет происходить".
Если же построить кротовую нору, если она возможно, и открыть её через 65 млн св.лет и мгновенно переместится к земле, не взирая на ограничение скорости, то переместимся к динозаврам.(ну или наоборот в будущее на 65 млн год.)

Можете обоссать только не бейте.
Аноним 01/10/18 Пнд 04:40:38 #503 №428070 
Вот космос исследуют и гаммой и видимым светом и микро и радио излучением, но были ли хотя бы разговоры о том что бы посмотреть сверх длинные волны непроницаемые для атмосферы? Или я чего-то не понимаю в приёмниках радиоизлучения?
Аноним 01/10/18 Пнд 05:49:54 #504 №428072 
>>427864
Они стесняются и терпят до дома, как хикки в школе.
Аноним 01/10/18 Пнд 05:54:45 #505 №428074 
>>427955
Ебнутые, ничего тайного. И секс в космосе был.
Аноним 01/10/18 Пнд 05:55:25 #506 №428075 
15323770831720.jpg
Давайте перекатимся, что ли.

>>428073 (OP)
>>428073 (OP)
>>428073 (OP)
>>428073 (OP)
Аноним 01/10/18 Пнд 09:10:57 #507 №428091 
>>428062
Гугли принцип причинности
Аноним 01/10/18 Пнд 12:50:13 #508 №428100 
>>428074
>Ебнутые, ничего тайного. И секс в космосе был
Между космонавтами и китайским модулем?
Аноним 01/10/18 Пнд 12:53:36 #509 №428101 
>>427961
>А ну личинус зародится, а ну как из-за невесомости он не сможет в земляшку, так и обречешь ЧИЛАВЕКА на мучительное существование! НИЛЬЗЯ ТАК ЭТАЖИРИБЕНОК!
Ну вот кстати лягушку-таки выращивали. Бедолага не знала где низ, а где вверх и тупо крутилась вокруг своей оси. Все время.
Аноним 01/10/18 Пнд 21:57:00 #510 №428202 
>>428100
Между >>428074 и его правой рукой в павильоне ВДНХ "Космос".
Аноним 02/10/18 Втр 12:02:17 #511 №428285 
>>424092
30 см на местности чтобы читалось. Мне ну нужно 3d и разные стороны, я буду делать дешифрирование.
Аноним 05/10/18 Птн 20:10:42 #512 №429460 
SpaceOnameetingtolight02341423.jpg
Пацаны, есть вопрос. Вот Солнце, оно светит, да? И типа там ядерные реакции идут, хуё-моё, масса превращается в энергию, разогревает внешние слои и хуярит излучением миллионы тонн вещества в пространство.

Так вот, если Солнце всё время становится легче, да ещё и отталкивает планеты от себя солнечным ветром, то почему планеты за миллиарды лет не разлетелись во все стороны? Орбиты же должны становиться выше. В целом хуйня же какая-то получается, а не солнечная система.
Аноним 05/10/18 Птн 20:29:22 #513 №429463 
>>427939
Это не полная, в полной есть знак ±. Угадай куда его нужно поставить ;-)
comments powered by Disqus

Отзывы и предложения